Prelims Cases
Prelims Cases
The mere fact that a title was issued by the Director of Lands does not confer
A. Governing Law any validity on such title if the property covered by the title or patent is part
B. Historical Background of the public forest.
C. Policy Considerations
D. Regalian Doctrine
Republic v Sayo
Sunbeam Convenience Foods, Inc. vs. CA FACTS: The case at bar started at 1961 whe the spouses Casiano and Luz
FACTS: Sunbeam Convenience Foods, Inc. is the recipient of a Sales Patent Sandoval filed an application for a parcel of land, Lot 7454 originally party of
issued by the Bureau of Lands over two parcels of land in Bataan. An OCT Santiago but had since then been transferred to Nueva Vizcaya.
was thereby issued. The Solicitor-General filed an action for reversion on the
ground that the lots were forest lands and therefore inalienable. The registration was opposed by Bayaua, Reyes, and the Philippine Cacao
and Farm Products.
CA ruled, upholding the Solicitor-General's contention.
The case went on until on 1981, 20 years after, the Heirs of Sandoval, Heirs of
ISSUE: Whether or not land is alienable Bayaua, and the Bureau of Lands and Bureau of Forest Development entered
a compromise agreement, which effectively distributed parts of lot 7454
HELD: The SC affirmed. among the aforesaid parties and the counsel of the Heirs of Sandoval as
attorney's fees. The compromise agreement was approved by the court and
Our adherence to the Regalian Doctrine subjects all agricultural, timber, and confirmed the title and ownership of the parties in accordance with its terms.
mineral lands to the dominion of the State. Thus, before any land may be
declassified from the forest group and converted into alienable or disposable Having knowledge of the incident, the Solicitor General filed a complaint
land for agricultural purposes, there must be a positive act from the before the court to annul the decision rendered by the court a quo for being
Government. Even rules on the confirmation of imperfect titles do not apply void and made in excess of jurisdiction or with grave abuse of discretion.
unless and until the land classified as forest land is released in an official
2 Tacder NatRes
The Solicitor General contended that the the Heirs of Sandoval et. al. did not In the proceeding at bar, it appears that the principal document relied upon
present any evidence to support their claims of ownership or registration, and presented by the applicants for registration, to prove the private
nor did the government agencies involve have a y authority to enter into the character of the large tract of land subject of their application, was a
compromise agreement, and finally, that he was not notified of the photocopy of a certification of the National Library. But, as this Court has
proceedings and so had not opportunity to take part therein. already had occasion to rule, that Spanish document cannot be considered
a title to property, it not being one of the grants made during the Spanish
As for the Heirs of Sandoval et.al.'s contention, they asseverate that the land regime, and obviously not constituting primary evidence of ownership. 6 It is
is not a public land as the possessory information title in their name and of an inefficacious document on which to base any finding of the private
their predecessors-in-interest, the pre-war certification appearing in the character of the land in question.
Bureau of Archives, and the fact that the proceeding of the registration was
brought under the Torrens act which presupposes an existing title to be It thus appears that the decision of the Registration Court a quo is based
confirmed, are all evidences that the land is a private land. solely on the compromise agreement of the parties. But that compromise
agreement included private persons who had not adduced any competent
ISSUE: W/N the respondent's evidences can be considered as proof that the evidence of their ownership over the land subject of the registration
lot 7454 is a private land. proceeding. Portions of the land in controversy were assigned to persons or
entities who had presented nothing whatever to prove their ownership of
RULING: NO. Under the Regalian Doctrine 2 all lands not otherwise any part of the land. The assent of the Directors of Lands and Forest
appearing to be clearly within private ownership are presumed to belong to Development to the compromise agreement did not and could not supply
the State. Hence it is that all applicants in land registration proceedings have the absence of evidence of title required of the private respondents.
the burden of overcoming the presumption that the land thus sought to be
registered forms part of the public domain. 3 Unless the applicant succeeds As to the informacion posesoria invoked by the private respondents, it
in showing by clear and convincing evidence that the property involved was should be pointed out that under the Spanish Mortgage Law, it was
acquired by him or his ancestors either by composition title from the Spanish considered a mode of acquiring title to public lands, subject to two (2)
Government or by possessory information title, or any other means for the conditions: first, the inscription thereof in the Registry of Property, and
proper acquisition of public lands, the property must be held to be part of second, actual, public, adverse, and uninterrupted possession of the land for
the public domain . 4 The applicant must present competent and persuasive twenty (20) years (later reduced to ten [10] years); but where, as here, proof
proof to substantiate his claim; he may not rely on general statements, or of fulfillment of these conditions is absent, the informacion posesoria cannot
mere conclusions of law other than factual evidence of possession and title. 5 be considered as anything more than prima facie evidence of possession. 7
3 Tacder NatRes
Finally, it was error to disregard the Solicitor General in the execution of the
compromise agreement and its submission to the Court for approval. It is, RULING No. Petitioners’ title is not valid.
after all, the Solicitor General, who is the principal counsel of the Under the Regalian Doctrine, all lands of public domain belong to the state,
Government; this is the reason for our holding that "Court orders and which the source of any is asserted right to ownership of land. All lands not
decisions sent to the fiscal, acting as agent of the Solicitor General in land otherwise appearing to be clearly within ownership are presumed to belong
registration cases, are not binding until they are actually received by the to the state.
Solicitor General." Although it may be true that Section 48, Chapter VIII of the Public Land Act
provides that those who by themselves or through their predecessor-in-
interest have been in open, continuous, exclusive, and notorious possession
Seville vs National Development Company and occupation of agricultural lands of the public domain for at least 30 years
FACTS: On January 14, 1980, Calixtra Yap sold to LSBDA a certain parcel of could acquire a title thereto, Section 4 of PD 1073 amended the same to limit
lot. LSBDA filed a Miscellaneous Sales Application with the Bureau of Lands its application to alienable and disposable lands of the public domain only.
covering said lot together with other lots acquired by them. After due notice In the case at bar, the Supreme Court reiterated that “unless public land is
and investigation, a Miscellaneous Sales Patent was issued in the name of shown to have been reclassified or alienated to a private person by the state,
LSBDA, on the basis of which, an original certificate of title was transcribed in it remains part of the inalienable public domain. Indeed, occupation thereof
the registration book on their name. in the concept of owner, no matter how long, cannot ripen into ownership
LSBDA subsequently assigned all its rights over the property to National and be registered as a title.
Development Company. NDC leased the property to Philippine Associated Absent any showing that the land has been classified as alienable, their
Smelting and Refining Corporation, PHILPHOS and Lepanto Consolidated possession thereof, no matter how lengthy, cannot ripen into ownership. In
Mining. other words, the petitioners have not become the owners of the disputed
On November 29, 1988, the estate of Joaquin Ortega represented by judicial property.
administrator Felipe Seville filed a complaint for recovery of real property, LSBD, who acquired a Miscellaneous Sales Patent over the subject property,
rentals and damages against the National Development Company. They which eventually caused them to have an Original Certificate of Title for the
argued that they acquired title to the disputed property by acquisitive said land has a valid claim over the property.
prescription, because they and their predecessors in interest had been in
possession of it for more than thirty years.
Director of Lands vs Intermediate Appellate Court
ISSUE: WON petitioners’ title, allegedly acquired by them through acquisitive FACTS:
prescription, is valid.
4 Tacder NatRes
Acme Plywood & Veneer Co., Inc., a corp. represented by Mr. Rodolfo it had already ceased to be of the public domain and had
Nazario, acquired from Mariano and Acer Infiel, members of the become private property, at least by presumption
Dumagat tribe 5 parcels of land The application for confirmation is mere formality, the lack
possession of the Infiels over the landdates back before the Philippines of which does not affect the legal sufficiency of the title as
was discovered by Magellan would be evidenced by the patent and the Torrens title to
land sought to be registered is a private land pursuant to RA 3872 be issued upon the strength of said patent.
granting absolute ownership to members of the non-Christian Tribes The effect of the proof, wherever made, was not to confer
on land occupied by them or their ancestral lands, whether with the title, but simply to establish it, as already conferred by the
alienable or disposable public land or within the public domain decree, if not by earlier law
Acme Plywood & Veneer Co. Inc., has introduced more than P45M 2. NO
worth of improvements If it is accepted-as it must be-that the land was already
ownership and possession of the land sought to be registered was duly private land to which the Infiels had a legally sufficient and
recognized by the government when the Municipal Officials of transferable title on October 29, 1962 when Acme acquired
Maconacon, Isabela it from said owners, it must also be conceded that Acme had
donated part of the land as the townsite of Maconacon Isabela a perfect right to make such acquisition
IAC affirmed CFI: in favor of The only limitation then extant was that corporations could
not acquire, hold or lease public agricultural lands in excess
ISSUES: of 1,024 hectares
1. W/N the land is already a private land - YES
2. W/N the constitutional prohibition against their acquisition by private
corporations or associations applies- NO
HELD: IAC affirmed Acme Plywood & Veneer Co., Inc United Paracale vs Dela
1. YES FACTS: United Paracale Mining Company, Inc. and Coco Grove, Inc., seek to
already acquired, by operation of law not only a right to a set aside the Order of dismissal of the case they filed with the trial court for
grant, but a grant of the Government, for it is not necessary
the ejectment of their respective defendants from the mining claims which
that a certificate of title should be issued in order that said
grant may be sanctioned by the courts, an application
therefore is sufficient
5 Tacder NatRes
were allegedly privately owned by them having been located and perfected however, does not amount to any vested right which could be the basis for
under the provisions of the Philippine Bill of 1902 and Act No. 624. their cause of action against herein private respondents.
Petitioners question the constitutionality of P.D. 1214 and unless it is What is precisely waived is their right to the issuance of a mining patent upon
successfully assailed, the petitioners will be but mere applicants for the lease application. This in effect grants the government the power, in the exercise
of the mining claims involved and would thus have no causes of action of its sound discretion, to award the patent to the applicant most qualified
against private respondents. to explore, develop and exploit the mineral resources of the country in line
with the objectives of P.D. 463, and not necessarily to the original locator of
ISSUE: Does the location and perfection of the mining claims under the
the mining claim. To sustain their contention that they can question the
provisions of the Philippine Bill of 1902 and Act No. 624 grant ownership to
award of mining patents to applicants other than them would put to naught
the petitioners.
the objectives of P.D. 1214
HELD: No. In the case of Santa Rosa Mining Company, Inc. vs. Leido, Jr. 11,
Republic vs Register of Deeds of Quezon
the court said that mere location does not mean absolute ownership over
Regalian Doctrine
the affected land or the mining claim. It merely segregates the located land
Burden of Proof of private ownership rests on plaintiff
or area from the public domain by barring other would-be locators from Doctrine of indefeasibility of Torrens title, exception
locating the same and appropriating for themselves the minerals found
therein. To rule otherwise would imply that location is all that is needed to FACTS: Petitioner was awarded a 17-hectare parcel of land, by virtue of which
acquire and maintain rights over a located mining claim. he was issued an OCT.
Moreover, the court does not agree with petitioners’ contention that they Through an investigation conducted by the Bureau of Lands, it was found
that the free patent acquired by Petitioner was fraudulent. A case for
cannot be placed in equal footing with those who forfeit all rights by virtue
falsification of public documents was filed by Petitioner was acquitted of the
of non-filing of an application within the prescribed period under PD1214.
crime.
The court said that while those who filed their mining lease applications have
better rights than those who forfeited all their right by not filing at all, this, Subsequently, the Solicitor-General filed a complaint against Petitioner,
praying for the declaration of nullity of the Free Patent and the OCT.
6 Tacder NatRes
Petitioner's main contention was that the land in question was no longer Moreover, the fact the Petitioner acquired a title to the land is of no moment,
within the unclassified public forest land because by the approval of his notwithstanding the indefeasibility of title issued under the Torrens System.
application for Free Patent by the Bureau of Lands, the land was already The indefeasibility of a certificate of title cannot be invoked by one who
alienable and disposable public agricultural land. He also claimed that the procured the same by means of fraud. Fraud here means actual and extrinsic
land was a small portion of Lot 5139, an area which had been declared -- an intentional omission of fact required by law.
disposable public land by the cadastral court.
Petitioner committed fraud by his failure to state that the land sought to be
ISSUE: Whether or not the land is alienable and disposable public land registered still formed part of the unclassified forest lands.
HELD: Under the Regalian Doctrine, all lands not otherwise clearly appearing Ituralde vs Falcasantos
to be privately-owned are presumed to belong to the State. Forest lands, like FACTS: In 1986, Ramon Ituralde obtained a 6k ha parcel of land in Basilan
mineral or timber lands which are public lands, are not subject to private Province. Falcasantos, meanwhile, applied with the Bureau of Lands to grant
ownership unless they under the Constitution become private properties. In him the same parcel of land under free patent. In 1989 the Director of Lands
the absence of such classification, the land remains unclassified public land allowed Ituralde to file a public land application for the subject property. 1990
until released therefrom and rendered open to disposition. the trial court named Ituralde the owner and possessor of the land. The CA
reversed this decision and set aside the trial court’s decision in Ituralde’s favor
The task of administering and disposing lands of the public domain belongs as the land was found to be forest land, and therefore, inalienable. Ituralde,
to the Director of Lands, and ultimately the Secretary of Agriculture and in his appeal to the SC, claims that since the Director of Lands allowed him
Natural Resources. Classification of public lands is, thus, an exclusive to file a public land application, the said land was no longer part of public
prerogative of the Executive Department, through the Office of the President. domain.
Courts have no authority to do so.
ISSUE: WON the filing of the public land application in 1989 made the land
Thus, in controversies involving the disposition of public agricultural lands, alienable. NO.
the burden of overcoming the presumption of state ownership of lands of
the public domain lies upon the private claimant. HELD: The CA is their decision found that since 1951, the subject parcel of
land was classified as a Forest Reserve Area, and hence, not capable of
In the present case, Petitioner failed to present clear, positive and absolute private appropriation and occupation. Before any land may be declassified
evidence to overcome said presumption and to support his claim. from the forest group and converted into alienable or disposable land for
7 Tacder NatRes
Republic vs. Court of Appeals and dela Rosa Under the theory of the respondent court, the surface owner will be planting
Facts: Jose dela Rosa filed an application for registration of a parcel of land on the land while the mining locator will be boring tunnels underneath. The
on his own behalf and on behalf of his children. This application was farmer cannot dig a well because he may interfere with the operations below
separately opposed by Benguet Consolidated, Inc. (Benguet) and Atok Big and the miner cannot blast a tunnel lest he destroy the crops above. How
Wedge Corporation (Atok). deep can the farmer, and how high can the miner, go without encroaching
on each other's rights? Where is the dividing line between the surface and
The petitioners claimed that they have acquired the land from their parents the sub-surface rights?
and that they have been in possession of the land ever since. Benguet and
Atok opposed on the ground that they have mineral claims covering the It is a well-known principle that the owner of piece of land has rights not only
property and had been in actual, continuous and exclusive possession of the to its surface but also to everything underneath and the airspace above it up
land in concept of owner. to a reasonable height.
The trial court denied the application while the Court of Appeals reversed The rights over the land are indivisible and that the land itself cannot be half
the decision of the trial court and recognized the claims of the applicant but agricultural and half mineral. The classification must be categorical; the land
subject to the rights of Benguet and Atok respecting their mining claims. In must be either completely mineral or completely agricultural.
other words, the Court of Appeals affirmed the surface rights of the de la
Rosas over the land while at the same time reserving the sub-surface rights The Regalian doctrine which, as its name implies, is intended for the benefit
of Benguet and Atok by virtue of their mining claims. of the State, not of private persons. The rule simply reserves to the State all
minerals that may be found in public and even private land devoted to
Issue: Whether or not the CA's ruling was correct. "agricultural, industrial, commercial, residential or (for) any purpose other
8 Tacder NatRes
than mining." Thus, if a person is the owner of agricultural land in which domain. It was only almost after two decades later or on 5 October 1953 that
minerals are discovered, his ownership of such land does not give him the the State ceded its right over the land in favor of the Spouses Gozo by
right to extract or utilize the said minerals without the permission of the State granting their patent application and issuing an original certificate of title in
to which such minerals belong. their favor. Prior to such conferment of title, the Spouses Gozo possessed no
The flaw in the reasoning of the respondent court is in supposing that the right to dispose of the land which, by all intents and purposes, belongs to
rights over the land could be used for both mining and non-mining purposes the State.
simultaneously. The correct interpretation is that once minerals are
discovered in the land, whatever the use to which it is being devoted at the Under the Regalian doctrine, which is embodied in Article XII, Section 2 of
time, such use may be discontinued by the State to enable it to extract the our Constitution, all lands of the public domain belong to the State, which is
minerals therein in the exercise of its sovereign prerogative. The land is thus the source of any asserted right to any ownership of land. All lands not
converted to mineral land and may not be used by any private party, appearing to be clearly within private ownership are presumed to belong to
including the registered owner thereof, for any other purpose that will the State. Accordingly, public lands not shown to have been reclassified or
impede the mining operations to be undertaken therein. The Regalian released as alienable agricultural land or alienated to a private person by the
doctrine then extends not only to land but also to “all natural wealth that State remain part of the inalienable public domain.
may be found in the bowels of the earth.”
The classification of public lands is an exclusive prerogative of the executive
department of the government and not the Courts. In the absence of such
Heirs of Gozo v. Philippine Union Mission Corp classification, the land remains as an unclassified land until it is released
Facts: Petitioners claim that they are the heirs of Spouses Gozo. The therefrom and rendered open to disposition. This is in consonance with the
respondents claim that they own a 5,000 square-meter portion of the Regalian doctrine that all lands of the public domain belong to the State and
property. The assertion is based on the 28 February 1937 Deed of Donation that the State is the source of any asserted right to ownership in land and
in favor of respondent Philippine Union Mission Corporation of the Seventh charged with the conservation of such patrimony.
Day Adventist (PUMCO-SDA). On the date the Deed of Donation is executed
in 1937, the Spouses Gozo were not the registered owners of the property All lands not appearing to be clearly within private ownership are presumed
yet although they were the lawful possessors thereof. The land in question to belong to the State. Accordingly, all public lands not shown to have been
was part of the inalienable public domain reclassified or released as alienable agricultural land or alienated to a private
person by the State remain part of the alienable public domain. As already
RULING: The Deed of Donation was executed by the Spouses Gozo on 28 well-settled in jurisprudence, no public land can be acquired by private
February 1937, the subject property was part of the inalienable public persons without any grant, express or implied, from the government; and it
9 Tacder NatRes
is indispensable that the person claiming title to public land should show that
Imperium vs Dominium
his title was acquired from the State or any other mode of acquisition
recognized by law. To prove that the land subject of an application for
registration is alienable, the applicant must establish the existence of a Lee Hong Kok vs. David
positive act of the government such as a presidential proclamation or an Distinction between IMPERIUM and DOMINIUM
executive order, an administrative action, investigation reports of Bureau of Only the government can question a void certificate of title issued pursuant
Lands investigators, and a legislative act or a statute. The applicant may also to a government grant.
secure a certification from the Government that the land applied for is
alienable and disposable. FACTS: This is regarding a piece of land which Aniano David acquired lawful
title thereto, pursuant to his miscellaneous sales application. After approval
It is beyond question that at the time the gratuitous transfer was effected by of his application, the Director of Lands issued an order of award and
the Spouses Gozo on 28 February 1937, the subject property was part of the issuance of sales patent, covering said lot by virtue of which the
public domain and is outside the commerce of man. It was only on 5 October Undersecretary of Agriculture and Natural Resources issued a Miscellaneous
1953 that the ownership of the property was vested by the State to the Sales Patent. The Register of Deeds then issued an original certificate of title
Spouses Gozo by virtue of its issuance of the OCT pursuant to the to David.
Homestead Patent granted by the President of the Philippines on 22 August
1953. Hence, the donation of the subject property which took place before 5 During all this time, Lee Hong Kok did not oppose nor file any adverse claim.
October 1953 is null and void from the very start.
ISSUE: Whether or not Lee Hong Kok may question the government grant
As a void contract, the Deed of Donation produces no legal effect
whatsoever. Quod nullum est, nullum producit effectum. That which is a HELD: Only the Government, represented by the Director of Lands or the
nullity produces no effect Logically, it could not have transferred title to the Secretary of Agriculture and Natural Resources, can bring an action to cancel
subject property from the Spouses Gozo to PUMCO-SDA and there can be a void certificate of title issued pursuant to a void patent. This was not done
no basis for the church's demand for the issuance of title under its name. by said officers but by private parties like the plaintiffs, who cannot claim that
Neither does the church have the right to subsequently dispose the property the patent and title issued for the land involved are void since they are not
nor invoke acquisitive prescription to justify its occupation. A void contract is the registered owners thereof nor had they been declared as owners in the
not susceptible to ratification, and the action for the declaration of absolute cadastral proceedings after claiming it as their private property.
nullity of such contract is imprescriptible.
10 Tacder NatRes
The fact that the grant was made by the government is undisputed. Whether Cruz vs Secretary of DENR
the grant was in conformity with the law or not is a question which the FACTS: Petitioners Isagani Cruz and Cesar Europa filed a suit for prohibition
government may raise, but until it is raised by the government and set aside, and mandamus as citizens and taxpayers, assailing the constitutionality of
the defendant cannot question it. The legality of the grant is a question certain provisions of Republic Act No. 8371, otherwise known as the
between the grantee and the government. Indigenous People’s Rights Act of 1997 (IPRA) and its implementing rules and
regulations (IRR). The petitioners assail certain provisions of the IPRA and its
IMPERIUM vs. DOMINIUM: IRR on the ground that these amount to an unlawful deprivation of the State’s
ownership over lands of the public domain as well as minerals and other
The government authority possessed by the State which is appropriately natural resources therein, in violation of the regalian doctrine embodied in
embraced int eh concept of sovereignty comes under the heading of section 2, Article XII of the Constitution.
imperium; its capacity to own or acquire property under dominium. The use
of this term is appropriate with reference to lands held by the State in its ISSUE: Do the provisions of IPRA contravene the Constitution?
proprietary character. In such capacity, it may provide for the exploitation
and use of lands and other natural resources, including their disposition, HELD: No, the provisions of IPRA do not contravene the Constitution.
except as limited by the Constitution. Examining the IPRA, there is nothing in the law that grants to the ICCs/IPs
ownership over the natural resources within their ancestral domain.
Ownership over the natural resources in the ancestral domains remains with
the State and the rights granted by the IPRA to the ICCs/IPs over the natural
resources in their ancestral domains merely gives them, as owners and
IPRA Law occupants of the land on which the resources are found, the right to the
1. Ancestral Lands and small scale utilization of these resources, and at the same time, a priority in
3. Native Title: Exception to of the public domain. They are private lands and belong to the ICCs/IPs by
Jura Regalia native title, which is a concept of private land title that existed irrespective of
any royal grant from the State. However, the right of ownership and
possession by the ICCs/IPs of their ancestral domains is a limited form of
ownership and does not include the right to alienate the same.
11 Tacder NatRes
Held : Yes
Cariño vs Insular Government The US Supreme Court reversed the decision of the Philippine Supreme
Facts: Mateo Carino(appellant) filed his petition in the Court of Land Court.
Registration to be granted a parcel of land consisting of 40 Hectares,1are and Mateo Carino should be granted what he seeks and should not be deprived
13 Centares in Baguio, Province of Benguet together with a house erected of what by the practice and belief of those among whom he lived, was his
thereon . It was granted , but the Government of the Philippines and also on property, through a refined interpretation of an almost forgotten law of
behalf of the United States averred having taken possession of the property Spain.
for public and military purposes, Respondents also asserted that they had The grant to the plaintiff was the result of the principle of Prescription as
title to all the land in the Philippines except to permit private lands to be mentioned in the royal cedula of 1754 states : “ Where such possessors shall
acquired and no prescription runs against the Spanish crown. The US not be able to produce title deeds, it shall be sufficient if they s hall show
succeeded the title of Spain through Treaty of Paris and Mateo’s land was that ancient possession, as a valid title by prescription” . Moreover, the
not registered and that he had lost all his rights and now is deemed to be a Decree of June 25, 1880 states that possessors for certain time shall be
mere trespasser. Then the Court of First Instance dismissed the deemed owners, if a cultivated land 20 years, if uncultivated 30 years.
application for Mateo since he did not possessed the land since time Mateo’s father was the owner of the land by the very terms of this decree-
immemorial and the land was property of the Government. The decision by Organic Act of July 1, 1902 ,all the property and rights acquired there by
was affirmed by the Philippine Supreme Court. Thus the case was brought the United States are to be administered “for the benefit of the inhibitants
to the US Supreme Court by virtue of Writ of Error(general method of thereof.”
bringing cases to this court, an appeal the exception, confined to equity in
the main.
Reservations of lands of the public domain are
valid assertions of Regalian Right
Mateo Carino in his appeal stated that he is an Igorot of the Province of
Benguet, they have owned the land for more than 50 years before the Treaty
of Paris ,they all had been recognized as owners by the Igorots and had Acting Registrars of Land Titles and Deeds of Pasay City vs. Regional Trial
inherited or received his land from his father in accordance with Igorot Court Branch 57 Makati
custom. However, no document of title had been issued from the Spanish FACTS:
Crown On November 5, 1985, Domingo Palomares, as administrator of the heirs of
Delfin Casal, commenced suit with the Regional Trial Court, Branch 132,
Issue : WON Carino owns the land. Makati, Metro Manila for declaratory relief, quieting of title, cancellation of
12 Tacder NatRes
Transfer Certificate of Title No. 192, and cancellation of entries upon Original
Certificate of Title No. 291. Civil Law; Ownership; Public Land; Presumption of ownership issued by right
of sovereignty.·Proclamation No. 192 (“RESERVING FOR THE VETERANS
However, during the trial the court found hard evidence on record that: (1) CENTER SITE PURPOSES CERTAIN PARCEL OF LAND OF THE PUBLIC
the property covered by OCT No. 291 had been conveyed to the United DOMAIN SITUATED IN THE PROVINCE OF RIZAL, ISLAND OF LUZON”) and
States of America; (2) it had been later ceded to the Republic of the Proclamation No. 423 (“RESERVING FOR MILITARY PURPOSES CERTAIN
Philippines; and (3) as a consequence, OCT No. 291 was cancelled upon final PARCELS OF THE PUBLIC DOMAIN SITUATED IN THE MUNICIPALITY OF
orders of Judge Ostrand. PASIG, TAGUIG, AND PARAÑAQUE, PROVINCE OF RIZAL, AND PASAY CITY”)
have the character of official assertions of ownership, and the presumption
During the pendency of the case also, Proclamation No. 192 (“RESERVING is that they have been issued by right of sovereignty and in the exercise of
FOR THE VETERANS CENTER SITE PURPOSES CERTAIN PARCEL OF LAND OF the State’s dominical authority. We take not only judicial notice thereof but
THE PUBLIC DOMAIN SITUATED IN THE PROVINCE OF RIZAL, ISLAND OF accept the same as a valid asseveration of regalian right over property
LUZON”) and Proclamation No. 423 (“RESERVING FOR MILITARY PURPOSES
CERTAIN PARCELS OF THE PUBLIC DOMAIN SITUATED IN THE Owners of property over which reconveyance is asserted are indispensable
MUNICIPALITY OF PASIG, TAGUIG, AND PARAÑAQUE, PROVINCE OF RIZAL, parties. While there is no presumption that property is Government property
AND PASAY CITY”) were issued by the government. until otherwise shown, because the law recognizes private ownership, thus:
Art. 425. Property of private ownership, besides the patrimonial property of
ISSUE: the State, provinces, cities, and municipalities, consists of all property
belonging to private persons, either individually or collectively. We find hard
Whether or not, Proclamation 192 and 423 were valid in the absence evidence on record that: (1) the property covered by OCT No. 291 had been
of proof of ownership of the property by the government? conveyed to the United States of America; (2) it had been later ceded to the
Republic of the Philippines; and (3) as a consequence, OCT No. 291 was
HELD: cancelled upon final orders of Judge Ostrand.
ISSUE: Were the Original Certificate of Titles issued to the petitioners valid?
Classification of Lands –NO
a) Constitution (Sec 3-5 Art XII)
b) Public Land Act
c) Classification of Public Lands Open to Disposition HELD Before the Treaty of Paris in 1899, the lands, whether agricultural,
d) Classification of Lands an Executive Privilege
e) Public Land vs Government mineral, or forest were under the exclusive patrimony and dominion of the
Spanish crown.Private ownership of land could only be acquired through
royal concessions which were documented in various forms, such as (1) Titulo
Real or Royal Grant," (2) Concession Especial or Special Grant, (3) Titulo de
Constitution (Sec 3-5 Art XII) Compra or Title by Purchase and (4) Informacion Posesoria or Possessory
Information title obtained under the Spanish Mortgage Law or under the
Palomo v. Court of Appeals Royal Decree of January 26, 1889.
FACTS: In 1913, some 440,530 square meters of land in Albay were reserved
for provincial park purposes by virtue of EO No. 40. Of said area, 15 parcels The decision of the CFI relied upon by petitioners were not signed by the
of land were registered in the name of Diego Palomo by the Court of First judge but were merely certified copies of notification to Diego Palomo
Instance. bearing the signature of the Clerk of Court.
In 1937, Diego Palomo donated these lands to his heirs Ignacio and Carmen More importantly, the lands in question were not classified as alienable lands.
Palomo. Claiming that the aforesaid original certificates of title were lost Since the lands were made part of a reservation for provincial park purposes,
during the Japanese occupation, Ignacio Palomo filed a petition for they form part of the forest zone. Thus, they cannot be the valid subject of
reconstitution with the Court of First Instance of Albay on May 30, 1950. The alienation.
Register of Deeds of Albay issued Transfer Certificates of Title Nos. 3911, 3912,
3913 and 3914 sometime in October 1953.
Republic vs Candy Maker Inc
On July 10, 1954, President Magsaysay issued Proclamation No. 47 converting FACTS: On April, 29, 1999, Antonia, Eladia, and Felisa, all surnamed Cruz,
the area covered by EO 40 into the Tiwi Hot Spring National Park. The executed a Deed of Absolute Sale in favor of Candy Maker, Inc. for a parcel
Palomos contended that they have been in possession of the subject lands of land located below the reglementary lake elevation of 12.50m, about 900
and have introduced improvements thereon. meters away the Laguna de Bay. Candy Maker, Inc. as applicant, filed an
application with the MTC of Taytay, Rizal for registration of its alleged title
over the lot. The CENRO of Antipolo City declared the land to fall within the
14 Tacder NatRes
alienable and disposable zone. On the other hand, the Land Registration FACTS: President Marcos through a presidential decree created PEA,
Authority recommended the exclusion of lot no. 3138-B on the ground that which was tasked with the development, improvement, and acquisition,
it is a legal easement and intended for public use, hence, inalienable and lease, and sale of all kinds of lands. The then president also transferred to
indisposable. On July 2001, the Republic of the Philippines, the Laguna Lake PEA the foreshore and offshore lands of Manila Bay under the Manila-Cavite
Development Authority (LLDA) filed its opposition which alleged that the lot Coastal
subject of the application for registration may not be alienated and disposed Road and Reclamation Project.
since it is considered part of the Laguna Lake Bed, a public land within, its
jurisdiction. Thereafter, PEA was granted patent to the reclaimed areas of land and
then, years later, PEA entered into a JVA with AMARI for the development of
ISSUE: Whether the property subject of the amended application is alienable the Freedom Islands. These two entered into a joint venture in the
and disposable property of the State, and if so, whether respondent adduced absence of any public bidding.
the requisite quantum of evidence to prove its ownership over the property?
Later, a privilege speech was given by Senator President Maceda
Held: The property subject of this application was alienable and disposable denouncing the JVA as the grandmother of all scams. An investigation was
public agricultural land. However, respondent failed to prove that it conducted and it was concluded that the lands that PEA was conveying to
possesses registrable title over the property. The statute of limitations with AMARI were lands of the public domain; the certificates of title over
regard to public agricultural lands does not operate against the statute the
unless the occupant proves possession and occupation of the same after a Freedom Islands were void; and the JVA itself was illegal. This prompted
claim of ownership for the required number of years to constitute a grant Ramos to form an investigatory committee on the legality of the JVA.
from the State. A mere casual cultivation of portions of the land by the
claimant does not constitute sufficient basis for a claim of ownership, such Petitioner now comes and contends that the government stands to
possession is not exclusive and notorious as to give rise to presumptive grant lose billions by the conveyance or sale of the reclaimed areas to
from the state. In light of the foregoing, the petition of the Republic of the AMARI. He also asked for the full disclosure of the renegotiations
Philippines is granted. happening between the parties.
ISSUE: W/N stipulations in the amended JVA for the transfer to AMARI
Chavez vs Public Estates Authority of the lands, reclaimed or to be reclaimed, violate the Constitution.
15 Tacder NatRes
The foreshore and submerged areas of Manila Bay are part of the lands of ISSUE:
the public domain, waters and other natural resources and consequently Did Lachica acquire the subject property by prescription?
owned by the State. As such, foreshore and submerged areas shall not be
alienable unless they are classified as agricultural lands of the public RULING:
domain. The mere reclamation of these areas by the PEA doesn’t convert The Court ruled on the negative.
these inalienable natural resources of the State into alienable and
disposable lands of the public domain. There must be a law or presidential Based from the facts and evidences presented, it was proven that Lachicha
proclamation officially classifying these reclaimed lands as alienable and only had a title to a 620sq.m. portion of the total area. Prescription cannot
disposable if the law has reserved them for some public or quasi- be had on the remaining area as the Court held that the law applicable in
public use. this case is Sec. 48 of CA 141 (wc deals with registration of lands of public
domain) and not Sec. 19 of Act 496 (wc deals with registration of private
lands) and with which the lower courts had relied on.
16 Tacder NatRes
occupation of the same under claim of ownership for the required number
The law in force at the time an action accrues is what governs the proceeding of years to constitute a grant from the State.
consistent with the fundamental dictum that laws shall have no retroactive
effect, unless the contrary is proved. In this case, the lower courts relied on Under (b), Sec. 48, CA 141, confirmation of an imperfect title to a public
the provisions on prescription with the assumption that the subject property domain requires that:
is a private land. However, the application for registration should be that of 1. There be an open, continuous, exclusive and notorious possession
a judicial confirmation of an imperfect title considering that the land is and occupation of agricultural lands of the public domain;
presumed under the Regalian Doctrine to be part of the public domain. 2. It should be under a bona fide claim of ownership; and
3. possession should be for at least thirty years immediately preceding
Public lands are classified into the filing of the application for confirmation of title except when
1. alienable or disposable lands which includes agricultural lands and prevented by war or force majeure
2. inalienable or non-disposable lands or those not susceptible of
private appropriation which includes Timber lands and Mineral In this case, Lachica had not yet satisfied the requirement of the 30 years
lands. For purposes of administration and disposition, the lands of possession, hence, prescription cannot be granted in favor of him.
the public domain classified as "disposable" or "alienable" are further
sub-classified into
a) Agricultural;
b) Residential, commercial, industrial or for similar productive
purposes;
Classification of Land an Executive Privilege
c) Educational, charitable or other similar purposes, and
d) Reservations for town sites and for public and quasi-public
purposes. Republic vs Imperial
FACTS: On September 12, 1917, the late Elias Imperial was issued Original
Possession of public agricultural land, however long the period may have Certificate of Title (OCT) 408 (500) pursuant to Decree No. 55173 of then
extended, never confers title thereto upon the possessor and it is because Court of First Instance of Albay. OCT No. 55173 was subdivided and further
the statute of limitations with regard to public agricultural land does not subdivided resulting in the issuance of several titles, which are now the
operate against the State, unless the occupant can prove possession and subjects of herein petition in the name of private respondents. Petitioner
Republic of the Philippines filed a case with the trial court to judicially declare
17 Tacder NatRes
the Transfer Certificates of Title (TCT) issued to herein private respondents HELD: Yes.
null and void on the ground that the subject land, on which the OCT was At the core of the controversy is whether the parcels of land in question are
based, has the features of a foreshore land based on an investigation foreshore lands. Foreshore land is a part of the alienable land of the public
conducted by the DENR, Region V, Legazpi City. Respondents, on the other domain and may be disposed of only by lease and not otherwise. It was
hand contend that Director of Lands found Jose Baritua's land covered by defined as "that part (of the land) which is between high and low water and
TCT No.18655, which stemmed from OCT 408(500), to be "definitely outside left dry by the flux and reflux of the tides." It is also known as "a strip of land
of the foreshore area." that lies between the high and low water marks and, is alternatively wet and
dry according to the flow of the tide."
Within the time for pleading, private respondents EANCRA Corporation,
Lolita Alcazar and Salvador Alcazar filed their answer with cross-claim, while The classification of public lands is a function of the executive branch of
the rest, namely, Felix S. Imperial, Feliza S. Imperial, Elias S. Imperial and government, specifically the director of lands (now the director of the Lands
Miriam S. Imperial filed a motion to dismiss. They contended that the Management Bureau). The decision of the director of lands when approved
adjudication by the cadastral court is binding against the whole world by the Secretary of the Department of Environment and Natural Resources
including the Republic since the cadastral proceedings are in rem and the (DENR) as to questions of fact is conclusive upon the court.
government itself through the Director of Lands instituted the proceedings
and was a direct and active participant therein. Petitioner, through the Office There is allegedly a conflict between the findings of the Director of Lands
of the Solicitor General, filed an objection to the motion to dismiss. After and the DENR, Region V, in the present case. Respondents contend that the
hearing the motion to dismiss, the trial court dismissed the complaint on the Director of Lands found Jose Baritua's land covered by TCT No.18655, which
ground that the judgment rendered by the cadastral court in G.R. Cad. Rec. stemmed from OCT 408(500), to be "definitely outside of the foreshore area."
No. 88 and the Courts resolution in the petition to quiet title, G.R. 85770, Petitioner, on the other hand, claims that subsequent investigation of the
both decreed that the parcel of land covered by OCT No. 408 (500) was not DENR, Region V, Legazpi City, disclosed that the land covered by OCT No.
foreshore. Petitioner appealed to the Court of Appeals. The appellate court 408 (500) from whence the titles were derived "has the features of a
denied petitioners motion for reconsideration for lack of merit and for failure foreshore land." The contradictory views of the Director of Lands and the
to file the appellants brief within the extended period granted to petitioner. DENR, Region V, Legazpi City, on the true nature of the land, which
contradiction was neither discussed nor resolved by the RTC, cannot be the
Hence, the present petition. premise of any conclusive classification of the land involved.
ISSUE: Whether or not the petition should be granted. The need, therefore, to determine once and for all whether the lands subject
of petitioner's reversion efforts are foreshore lands constitutes good and
18 Tacder NatRes
sufficient cause for relaxing procedural rules and granting the third and of the title of the lots in the names of the applicants, herein private
fourth motions for extension to file appellant's brief. Petitioner's appeal respondents. On appeal, the Appellate Court affirmed the trial court's
presents an exceptional circumstance impressed with public interest and decision.
must then be given due course.
Issue: Whether the lots in question may be registered under Section 48(b) of
In the case at bar, the need to determine once and for all whether the lands CA 141
subject of petitioners reversion efforts are foreshore lands constitutes good
and sufficient cause for relaxing the procedural rules and granting the third Ruling: In the case of Bureau of Forestry vs. Court of Appeals, 153 SCRA 351,
and fourth motions for extensions to file appellants brief. Petitioner Republics we ruled:
appeal presented an exceptional circumstance impressed with public interest
which in the Courts discretion must be given due course. "As provided for under Section 6 of Commonwealth Act 141, which was
lifted from Act 2874, the classification or reclassification of public lands into
alienable or disposable, mineral or forest lands is now a prerogative of the
Director of Lands vs CA and Bisnar Executive Department of the government and not the courts. With these
Facts: In their joint application for registration of title to two (2) parcels of rules, there should be no more room for doubt that it is not the court which
land filed on July 20, 1976, the applicants Ibarra and Amelia Bisnar claimed determines the classification of lands of the public domain into agricultural,
to be the owners in fee simple of Lots 866 and 870 of the Pilar Cadastra, forest or mineral but the Executive Branch of the government, through the
containing an area of 28 hectares (284,424 sq.m.) and 34 hectares (345,385 Office of the President. Hence, it was grave error and/or abuse of discretion
sq.m.) situated in barrio Gen. Hizon, Municipality of President Roxas, Province for respondent court to ignore the uncontroverted facts that (1) the
of Capiz (p. 14, Rollo). The applicants alleged that they inherited those parcels disputed area is within a timberland block, and (2) as certified to by the
of land (p. 41, Rollo) and they had been paying the taxes thereon (p. 40, then Director of Forestry, the area is needed for forest purposes." (pp. 21-
Rollo). The Director of Lands and the Director of the Bureau of Forest 22, Rollo.)
Development, opposed the application. On February 24, 1977, the applicants
filed an amended application, which was approved on March 14, 1977, and It bears emphasizing that a positive act of the government is needed to
included the following allegation: "Should the Land Registration Act invoked declassify land which is classified as forest and to convert it into alienable or
be not applicable to the case, they hereby apply for the benefits of Chapter disposable land for agricultural or other purposes (Republic vs. Animas, 56
8, Commonwealth Act 141, as amended, as they and their predecessors-in- SCRA 499). Unless and until the land classified as forest is released in an
interest have been in possession of the land as owners for more than fifty official proclamation to that effect so that it may form part of the disposable
(50) years." (p. 16, Rollo.) After hearing, the trial court ordered the registration
19 Tacder NatRes
agricultural lands of the public domain, the rules on confirmation of not the court which determines the classification of lands of the public
imperfect title do not apply. domain into agricultural, forest or mineral but the Executive Branch of the
government, through the Office of the President.
Thus, possession of forest lands, however long, cannot ripen into private
ownership. POSITIVE ACT OF THE GOVERNMENT NEEDED TO DECLASSIFY LAND INTO
ALIENABLE LAND FOR AGRICULTURAL OR OTHER PURPOSES. — It bears
Section 48 (b) of Commonwealth Act No. 141, as amended, applies exclusively emphasizing that a positive act of the government is needed to declassify
to public agricultural land. Forest lands or areas covered with forests are land which is classified as forest and to convert it into alienable or disposable
excluded (p. 26, Rollo). land for agricultural or other purposes (Republic v. Animas, 56 SCRA 499).
Unless and until the land classified as forest is released in an official
We reiterate our ruling in Amunategui that: "In confirmation of imperfect title proclamation to that effect so that it may form part of the disposable
cases, the applicant shoulders the burden of proving that he meets the agricultural lands of the public domain, the rules on confirmation of
requirements of Section 48, Commonwealth Act No. 141, as amended by imperfect title do not apply.
Republic Act 1942. He must overcome the presumption that the land he is
applying for is part of the public domain but that he has an interest therein 1. the classification or reclassification of public lands into alienable or
sufficient to warrant registration in his name because of an imperfect title disposable agricultural land, mineral land or forest land is a prerogative of
such as those derived from old spanish grants or that he has had continuous, the Executive Department of the government and not of the courts;
open and notorious possession and occupation of agricultural lands of the 2. that possession of forest lands, no matter how long, cannot ripen into
public domain under a bona fide claim of acquisition of ownership for at least private ownership; and
thirty (30) years preceding the filing of his application." (Heirs of Amunategui 3. that an applicant for registration of title has the burden of proving that he
vs. Director of Forestry, 126 SCRA 69.) meets the requirements of Section 48 of Com. Act No. 141, as amended.
Caloocan, used as a fishery having a superficial area of 10,805 square meters, Examples in point are properties occupied by public buildings or devoted to
and bounded as set out in the petition; its value according to the last municipal or other governmental uses.
assessment being $505.05, United States currency. This petition was opposed
by the Solicitor-General in behalf of the Director of Lands, and by the entity It is settled that the general legislation of Congress in respect to public lands
known asObras Pias de la Sagrada Mitra, the former on the ground that the does not extend to tide lands. It provided that the scrip might be located on
land in question belonged to the Government of the United States, and the the unoccupied and unappropriated public lands. As said in Newhall vs.
latter, that it was the absolute owner of all the dry land along the eastern Sanger (92 U.S. 761, 763.) A marshland which is inundated by the rise of tides
boundary of the said fishery. The Court of Land Registration in its decision of belong to the State and is not susceptible to appropriation by occupation,
December 1, 1906, dismissed the said oppositions without costs and decreed, has no application in the present case inasmuch as in said case the land
after a general entry by default, the adjudication and registration of the subject matter of the litigation was not yet titled
property described in the petition, in favor of Isabelo Montano y Marcial.
From this decision only counsel for the Director of Public Lands appealed to
this court. and precisely Isabelo Montano sought title thereon on the
strength of 10 years' occupation pursuant to paragraph 6, section 5 of Act Modes of Disposition
926 of the Philippine Commission a) Homestead Settlement
b) Sale of Public Agricultural Land
c) Lease
Issue: Whether or not the land in question can be acquired by Montano d) Confirmation of Imperfect Title
e) Non-registrable Land
f) Administrative Legalization
Held: Accordingly, "government land" and "public domain" are not g) Special Patents
synonymous items. The first includes not only the second, but also other
lands of the Government already reserved or devoted to public use or subject
to private right. In other words, the Government owns real estate which is
part of the "public lands" and other real estate which is not part thereof.
Government property was of two kinds — first, that of public use or service, Modes of Disposition
appropriate with reference to lands held by the state in its proprietary lands may remain free and unencumbered for us to dispose of as we may
character. In such capacity, it may provide for the exploitation and use of wish."
lands and other natural resources, including their disposition, except as
limited by the Constitution. Dean Pound did speak of the confusion that It could therefore be affirmed in Montano v. Insular Government that "as to
existed during the medieval era between such two concepts, but did note the unappropriated public lands constituting the public domain the sole
the existence of res publicae as a corollary to dominium. As far as the power of legislation is vested in Congress, . . ." They continue to possess that
Philippines was concerned, there was a recognition by Justice Holmes in character until severed therefrom by state grant. Where, as in this case, it
Cariño v. Insular Government, a case of Philippine origin, that "Spain in its was found by the Court of Appeals that the disputed lot was the result of
earlier decrees embodied the universal feudal theory that all lands were held reclamation, its being correctly categorized as public land is undeniable.
from the Crown . . ." What was held in Heirs of Datu Pendatun v. Director of Lands finds
application. Thus: "There being no evidence whatever that the property in
That was a manifestation of the concept of jura regalia, which was adopted question was ever acquired by the applicants or their ancestors either by
by the present Constitution, ownership however being vested in the state as composition title from the Spanish Government or by possessory information
such rather than the head thereof. What was stated by Holmes served to title or by any other means for the acquisition of public lands, the property
confirm a much more extensive discussion of the matter in the leading case must be held to be public domain." For it is well-settled "that no public land
of Valenton v. Murciano, decided in 1904. One of the royal decrees cited was can be acquired by private persons without any grant, express or implied,
incorporated in the Recopilacion de Leyes de las Indias in these words: "We from the government." It is indispensable then that there be a showing of a
having acquired in sovereignty over the Indies, and all lands, territories, and title from the state or any other mode of acquisition recognized by law. The
possessions not heretofore ceded away by our royal predecessors, or by us, most recent restatement of the doctrine, found in an opinion of Justice J.B.L.
or in our name, still pertaining to the royal crown and patrimony, it is our will Reyes, follows: "The applicant, having failed to establish his right or title over
that all lands which are held without proper and true deeds of grant be the northern portion of Lot No. 463 involved in the present controversy, and
restored to us according as they belong to us, in order that after reserving there being no showing that the same has been acquired by any private
before all what to us or to our viceroys, audiencias, and governors may seem person from the Government, either by purchase or by grant, the property
necessary for public squares, ways, pastures, and commons in those places is and remains part of the public domain." To repeat, the second assignment
which are peopled, taking into consideration not only their present condition, of error is devoid of merit.”
but also their future and their probable increase, and after distributing to the
natives what may be necessary for tillage and pasturage, confirming them in Director of Land vs CA and Valeriano
what they now have and giving them more if necessary, all the rest of said
23 Tacder NatRes
was, in fact. Since the subject property is still unclassified, whatever resolution 10-Mos. later reversing the decision of the Regional Trial Court of
possession Applicants may have had, and, however long, cannot ripen into Pasig City. The RTC decision confirmed the application for the titling of the
private ownership. (Adorable v. Director of Lands, 107 Phil. 401; Director of parcel of land with aggregate area of 2112 sqm located at Brgy Ususan,
Forestry v. Muñoz, 23 SCRA 1184-1216 [1968]; Director of Lands v. Abanzado, Taguig, Metro Manila, in favor of the Menguito’s.
65 SCRA 5 [1975]; Republic v. Court of Appeals, 89 SCRA 648, 656 [1979]).
The application of TCT was brought by the operation of the Land Registration
4. ID.; ID.; ID.; ID.; CADASTRAL SURVEY OF MUNICIPALITY DOES NOT of Authority as amended by the Property Registration Decree No. 1529
AUTOMATICALLY RELEASE ALL LANDS THEREIN AS ALIENABLE. — While it proceeding to declare:
may be that the Municipality of Obando has been cadastrally surveyed in
1961, it does not follow that all lands comprised therein are automatically a) That its applicants –the Menguito’s, are owners, in fee simple, these
released as alienable. A survey made in a cadastral proceeding merely 11 parcels of land.
identifies each lot preparatory to a judicial proceeding for adjudication of b) Listing the applicants lot title numbers, attendant documents given
title to any of the lands upon claim of interested parties. Besides, if land is with their respective Technical Descriptions.
within the jurisdiction of the Bureau of Forest Development, it would be
beyond the jurisdiction of the Cadastral Court to register it under the Torrens For the said application, the RTC of Pasig issued a notice of its initial hearing
System. against the whole world publishing the same at Abante Tabloid on April 5,
1989.
5. ID.; ID.; ID.; ID.; CONVERSION OF UNCLASSIFIED LAND INTO A FISHPOND
DOES NOT MAKE LAND ALIENABLE. — The conversion of subject property Seven days earlier, however, March 30, 1989, the Office of Solicitor General,
into a fishpond by Applicants, or the alleged titling of properties around it filed its contention as:
does not automatically render the property as alienable and disposable. 1. Applicants nor its predecessor’s in interest were neither in open,
Applicants’ remedy lies in the release of the property from its present continuous, exclusive, adverse and notorious possession or
classification. occupation of the land they applied for since 1945.
2. Applicants don’t have competent and sufficient evidence of bonafide
acquisition –without open, uninterrupted-continuous, exclusive,
adverse or notorious occupation of the lot in the concept of the
Menguito vs Republic owner and so appearing not genuine or indicative of pretended
FACTS: A petition for review assailing the Court of Appeals Sept 30, 1997 possession.
decision against the Menguito’s –the petitioners, with promulgated
25 Tacder NatRes
3. Titling from fee simple with Spanish grant title were not anymore Applicants to avail the OCT they must overcome the presumption
available after Feb 16, 1976 as required by PD 892. that the land they applied for forms part of the public domain and
4. That the said parcel of land applied for is part of the public domain if so, it has to be shown as re-classified or alienated to a private
and belonging to the Republic of the Philippines –and not subject to person by the state.
private appropriation. Otherwise, the land remain inalienable public domain.
To overcome this, an incontrovertible or clear and convincing
The OSG thus stated its valid opposition on the presented documents by the evidence must be presented.
applicants leaving its prayers that said application be denied and land in The court observed that applicants possession were tacked only as
question be reverted to the ownership of the Republic of the Philippines. At far back as 1948 to its predecessors of interest –the same must have
the appellate court, the RTC’s decision favoring the registrations of the land shown that their predecessors in interest were in possession of the
applied were reversed and thus this petition. property by some 30-years back or 1938 before WW-2.
ISSUE: WON the CA’s did not err in its decision to reverse the trial court The court finds the petitioners failed to show it even as they claimed that
findings. Cirilo Menguito once declared the land for tax purposes in 1943 –yet failed
to show documentary evidence to support the claim.
HELD: No. The petition is without merit. The court therefore find no reason to modify the appellate court’s decision -
thus denied the petition and declared cost against petitioner.
The Court cited Sec. 48 of Commonwealth Act No. 141 as amended, provide
the registration the registration of the title of lands in this wise; Note:
“Unless public land is shown to have been reclassified or alienated to a
“(b) Who by themselves, or their predecessors in interest have been in private person by the State, it remains part of the inalienable public domain.
open, continuous, exclusive, and notorious occupation of the land in the Indeed, occupation thereof in the concept of owner, no matter how long,
concept of the owner.” cannot ripen into ownership and be registered as a title.
The Court thus observed that the petitioners have insufficient evidence on
this.”
occupation of the land in question, first, by Apolonio Garcia and should be issued in order that said grant may be sanctioned by the
Basilio Mendoza, and then by Valentin Susi has been open, courts, an application therefore is sufficient, under the provisions of
continuous, adverse and public, without any interruption, except section 47 of Act No. 2874. If by a legal fiction, Valentin Susi had
during the revolution, or disturbance, except when Angela Razon acquired the land in question by a grant of the State, it had already
commenced an action in the Court of First Instance of Pampanga to ceased to be the public domain and had become private property,
recover the possession of said land. at least by presumption, of Valentin Susi, beyond the control of the
Having failed in her attempt to obtain possession of the land in Director of Lands. Consequently, in selling the land in question to
question through the court, Angela Razon applied to the Director of Angela Razon, the Director of Lands disposed of a land over which
Lands for the purchase thereof. he had no longer any title or control, and the sale thus made was
After making the proper administrative investigation, the Director of void and of no effect, and Angela Razon did not thereby acquire any
Lands overruled the opposition of Valentin Susi and sold the land to right.
Angela Razon and issued the proper certificate of title to Angela
Razon.
Angela Razon required Valentin Susi to vacate the land in question.
Issue:
Who is then the rightful owner of the land?
Held:
SC in their decision favoured Valentin Susi. According to SC there is,
the presumption juris et de jure established in paragraph (b) of
section 45 of Act No. 2874, amending Act No. 926, that all the
necessary requirements for a grant by the Government were
complied with, for he has been in actual and physical possession,
Sale of Public Agricultural Land
personally and through his predecessors, of an agricultural land of Lease
the public domain openly, continuously, exclusively and publicly Confirmation of Imperfect Title
since July 26, 1894, with a right to a certificate of title to said land
under the provisions of Chapter VIII of said Act. So that when Angela
Razon applied for the grant in her favor, Valentin Susi had already
acquired, by operation of law, not only a right to a grant, but a grant Alba vs CA
of the Government, for it is not necessary that certificate of title
28 Tacder NatRes
Facts: Applicant Jose Lachica filed this application for title to land on April 28, Alba, who purchased the same from Dionisia Regado in 1918. Hence, they
1958 with the claim that the land applied for was purchased by him and his have been in possession continuously, openly and peacefully under claim of
wife, Adela Raz from one Eulalio Raz. The documents attached to the ownership of the above-mentioned portion for not less 70 years.
application are: technical description, surveyor's certificate, certification by
the chief deputy assessor of Aklan and the blue print of Psu-161277.
The land applied for is residential and initial hearing was held on October 31, The private respondent/applicant requested the Municipal Assessor of
1958. An order of general default was issued but those who presented their Banga to issue a revised tax declaration covering 4,845 square meters on the
opposition, namely, Octabela Alba Vda. De Raz, Manuel and Susana Braulio, bare claim that "the area has been decreased" to only 620 square meters.
Jose Rago, representing Apolonia Rebeco, the Director of Lands and the
Municipality of Banga represented by the Provincial Fiscal, were given thirty Issue: Whether or not the private respondent/applicant is entitled to the
(30) days to file their written opposition. confirmation of his ownership in fee simple for the 4, 845 square meter parcel
Manuel C. Braulio and Susana P. Braulio filed their opposition on October 31, of land he applied for.
1958. They opposed the registration of the southeastern portion of the 240
square meters of the land applied for alleging that they are the owners in fee Held: The trial court and the Court of Appeals confirmed private
simple and possessors of said portion and all the improvements thereon for respondent/applicant's title to the land on the basis of the findings that:
not less than 70 years together with their predecessor-in-interest deriving 1. the private respondent/applicant purchased the land from Faustino
their title by purchase from the original owners. Martirez;
Jose Rago filed his opposition on November 29, 1958 as the duly constituted 2. the subject land is covered by Tax Declaration No. 14181;
attorney-in-fact of Apolonia Rebeco although no special power of attorney 3. the private respondent/applicant has paid the realty taxes on the
was attached. He alleged that his principal is the owner by right of succession land from 1945 up to the filing of his application in 1958;
and is in the possession of said portion with all its improvements for more 4. the private respondent/applicant has been in actual, open and
than 80 years together with his predecessor-in-interest, continuously, continuous possession of the subject land in the concept of owner
peacefully and openly under claim of ownership. since 1945, and
Rodolfo Alba, Lourdes Alba, represented by their attorney-in-fact, alleged 5. the private respondent/applicant has acquired the land by
that they are the co-owners of a portion of the land applied for with an area prescription.
of 2,262 square meters bounded on the north by Januario Masigon, Nicolas
Realtor, Agustina Rebeldia and Apolonia Rebeco, on the south by Eulalio Raz Private respondent that the acquired land in question from three (3) sources,
and on the west by the public market of Banga. They claimed to have namely:
inherited the above-mentioned portion from their late father, Eufrosino M.
29 Tacder NatRes
a) A Deed of Sale dated August 13, 1941 allegedly executed by Faustino when prevented by war or force majeure. These shall be conclusively
Martirez covering 840 square meters; presumed to have performed all the conditions essential to a
b) 300 square meters allegedly purchased from private respondent's Government grant and shall be entitled to a certificate of title under
father-in-law Eulalio Raz, and the provisions of this chapter. 50
c) 3,725 square meters private respondent allegedly bought in 1940 3. Members of the national cultural minorities who by themselves or
from Eugrocino Alba. through their predecessors-in-interest have been in open,
continuous, exclusive and notorious possession and occupation of
In Section 48 of Commonwealth Act 141, as amended by RA Nos. 1942 and lands of the public domain suitable to agriculture, whether
6236, 48 which states that: disposable or not, under a bona fide claim of ownership for at least
Sec. 48. The following-described citizens of the Philippines, occupying lands 30 years shall be entitled to the rights granted in subsection (b)
of the public domain or claiming to own any such lands or an interest therein, hereof. 51
but whose titles have not been perfected or completed, may apply to the A circumspect scrutiny of the assailed Decision readily shows that in the
Court of First Instance of the province where the land is located for affirming the ruling of the trial court, the Court of Appeals relied on the
confirmation of their claim and issuance of a certificate of title therefor, under provisions of Section 19 of Act 496 52 in relation to the Civil Code's provision's
the Land Registration Act, to wit: on prescription on the assumption that the subject land is private land.
1. Those who prior to the transfer of sovereignty from Spain to the Therein lies the flaw in the appellate court's postulate. The application for
United States have applied for the purchase, composition or other registration of private respondent is for judicial confirmation of an imperfect
form of grant of lands of the public domain under the laws and royal title considering that the land is presumed under the Regalian Doctrine to be
decrees then in force and have instituted and prosecuted the part of the public domain.
proceedings in connection therewith, but have with or without Public lands are broadly classified into 1.] Alienable or disposable lands; and,
default upon their part, or for any other cause, not received title 2.] Inalienable or non-disposable public lands. Non-disposable public lands
therefor, if such applicants or grantees and their heirs have occupied or those not susceptible of private appropriation include a.] Timber lands;
and cultivated said lands continuously since the filing of their and, b.] Mineral lands. 53 For purposes of administration and disposition, the
applications. 49 lands of the public domain classified as "disposable" or "alienable" are further
2. Those who by themselves or through their predecessors in interest sub-classified into a.] Agricultural; b.] Residential, commercial, industrial or
have been in open, continuous, exclusive and notorious possession for similar productive purposes; c.] Educational, charitable or other similar
and occupation of agricultural lands of the public domain under a purposes, and d.] Reservations for town sites and for public and quasi-public
bona fide claim of ownership, for at least thirty years immediately purposes. 54
preceding the filing of the application for confirmation of title except
30 Tacder NatRes
Rule: title in the name of Naguit upon failure of Rustico Angeles to appear during
The Decision of the Regional Trial Court of Kalibo, Aklan, Branch 1 dated trial after filing his formal opposition to the petition.
August 18, 1992 in Land Registration Case No. K-101, LRC Record No. K-15104
is hereby MODIFIED as follows: The Solicitor General, representing the Republic of the Philippines, filed a
1. The 620 square meter portion on which private respondent Jose N. motion for reconsideration on the grounds that the property which is in
Lachica's house is situated, clearly delineating its metes and bounds, open, continuous and exclusive possession must first be alienable. Naguit
is hereby ORDERED segregated from the parcel of land described in could not have maintained a bona fide claim of ownership since the subject
Psu-161277 situated in the Poblacion of the Municipality of Banga, land was declared as alienable and disposable only on October 15, 1980. The
Province of Aklan, Philippines with an area of 4,484 square meters, alienable and disposable character of the land should have already been
to be registered and confirmed in the name of private respondent; established since June 12, 1945 or earlier.
2. A ten (10) meter road width along the National road mentioned in
the application be segregated for future road widening programs ISSUE:
upon the payment of just compensation to be annotated at the back Whether or not it is necessary under Section 14 (1) of the Property
of the title. Registration Decree that the subject land be first classified as alienable and
3. Insofar as the ownership of the remainder of the subject land is disposable before the applicant’s possession under a bona fide claim of
concerned, the case is hereby REMANDED to the court of origin for ownership could even start.
the reception of further evidence for the petitioners to establish the
other requisites for the confirmation of title and registration in their RULING:
names of the areas they respectively claim. Section 14 (1) merely requires that the property sought to be registered as
already alienable and disposable at the time the application for registration
of title is filed.
Republic vs CA and Naguit
FACTS: Corazon Naguit filed a petition for registration of title which seeks There are three requirements for registration of title,
judicial confirmation of her imperfect title over a parcel of land in Nabas, 1. that the subject property is alienable and disposable;
Aklan. It was alleged that Naguit and her predecessors-in-interest have 2. that the applicants and their predecessor-in-interest have been in
occupied the land openly and in the concept of owner without any objection open, continuous, and exclusive possession and occupation, and;
from any private person or even the government until she filed her 3. that the possession is under a bona fide claim of ownership since
application for registration. The MCTC rendered a decision confirming the June 12, 1945.
31 Tacder NatRes
There must be a positive act of the government through a statute or On 20 February 1998, Mario Malabanan filed an application for land
proclamation stating the intention of the State to abdicate its exclusive registration before the RTC of Cavite-Tagaytay, covering a parcel of land
prerogative over the property, thus, declaring the land as alienable and situated in Silang Cavite, consisting of 71,324 square meters. Malabanan
disposable. However, if there has been none, it is presumed that the claimed that he had purchased the property from Eduardo Velazco, and that
government is still reserving the right to utilize the property and the he and his predecessors-in-interest had been in open, notorious, and
possession of the land no matter how long would not ripen into ownership continuous adverse and peaceful possession of the land for more than thirty
through acquisitive prescription. (30) years. Velazco testified that the property was originally belonged to a
twenty-two hectare property owned by his great-grandfather, Lino Velazco.
To follow the Solicitor General’s argument in the construction of Section 14 Lino had four sons– Benedicto, Gregorio, Eduardo and Esteban–the fourth
(1) would render the paragraph 1 of the said provision inoperative for it would being Aristedes’s grandfather. Upon Lino’s death, his four sons inherited the
mean that all lands of public domain which were not declared as alienable property and divided it among themselves. But by 1966, Esteban’s wife,
and disposable before June 12, 1945 would not be susceptible to original Magdalena, had become the administrator of all the properties inherited by
registration, no matter the length of unchallenged possession by the the Velazco sons from their father, Lino. After the death of Esteban and
occupant. In effect, it precludes the government from enforcing the said Magdalena, their son Virgilio succeeded them in administering the
provision as it decides to reclassify lands as alienable and disposable. properties, including Lot 9864-A, which originally belonged to his uncle,
Eduardo Velazco. It was this property that was sold by Eduardo Velazco to
The land in question was found to be cocal in nature, it having been planted Malabanan.
with coconut trees now over fifty years old. The inherent nature of the land
but confirms its certification in 1980 as alienable, hence agricultural. There is Among the evidence presented by Malabanan during trial was a Certification
no impediment to the application of Section 14 (1) of the Property dated 11 June 2001, issued by the Community Environment & Natural
Registration Decree. Naguit had the right to apply for registration owing to Resources Office, Department of Environment and Natural Resources
the continuous possession by her and her predecessors-in-interest of the (CENRO-DENR), which stated that the subject property was “verified to be
land since 1945. within the Alienable or Disposable land per Land Classification Map No. 3013
established under Project No. 20-A and approved as such under FAO 4-1656
on March 15, 1982.” On 3 December 2002, the RTC approved the application
for registration.
Heirs of Malabanan vs Republic
FACTS: The Republic interposed an appeal to the Court of Appeals, arguing that
Malabanan had failed to prove that the property belonged to the alienable
32 Tacder NatRes
and disposable land of the public domain, and that the RTC had erred in a) Since Section 48(b) merely requires possession since 12 June 1945
finding that he had been in possession of the property in the manner and for and does not require that the lands should have been alienable and
the length of time required by law for confirmation of imperfect title. On 23 disposable during the entire period of possession, the possessor is
February 2007, the Court of Appeals reversed the RTC ruling and dismissed entitled to secure judicial confirmation of his title thereto as soon as
the appliocation of Malabanan. it is declared alienable and disposable, subject to the timeframe
imposed by Section 47 of the Public Land Act.
b) The right to register granted under Section 48(b) of the Public Land
ISSUES: Act is further confirmed by Section 14(1) of the Property Registration
Decree.
1. In order that an alienable and disposable land of the public domain 2. For purposes of Section 14(2) of the Property Registration Decree
may be registered under Section 14(1) of Presidential Decree No. may a parcel of land classified as alienable and disposable be
1529, otherwise known as the Property Registration Decree, should deemed private land and therefore susceptible to acquisition by
the land be classified as alienable and disposable as of June 12, 1945 prescription in accordance with the Civil Code?
or is it sufficient that such classification occur at any time prior to the
filing of the applicant for registration provided that it is established In complying with Section 14(2) of the Property Registration Decree, consider
that the applicant has been in open, continuous, exclusive and that under the Civil Code, prescription is recognized as a mode of acquiring
notorious possession of the land under a bona fide claim of ownership of patrimonial property. However, public domain lands become
ownership since June 12, 1945 or earlier? only patrimonial property not only with a declaration that these are alienable
or disposable. There must also be an express government manifestation that
In connection with Section 14(1) of the Property Registration Decree, Section the property is already patrimonial or no longer retained for public service
48(b) of the Public Land Act recognizes and confirms that “those who by or the development of national wealth, under Article 422 of the Civil Code.
themselves or through their predecessors in interest have been in open, And only when the property has become patrimonial can the prescriptive
continuous, exclusive, and notorious possession and occupation of alienable period for the acquisition of property of the public dominion begin to run.
and disposable lands of the public domain, under a bona fide claim of a) Patrimonial property is private property of the government. The
acquisition of ownership, since June 12, 1945” have acquired ownership of, person acquires ownership of patrimonial property by prescription
and registrable title to, such lands based on the length and quality of their under the Civil Code is entitled to secure registration thereof under
possession. Section 14(2) of the Property Registration Decree.
33 Tacder NatRes
b) There are two kinds of prescription by which patrimonial property Neither can petitioners properly invoke Section 14(2) as basis for registration.
may be acquired, one ordinary and other extraordinary. Under While the subject property was declared as alienable or disposable in 1982,
ordinary acquisitive prescription, a person acquires ownership of a there is no competent evidence that is no longer intended for public use
patrimonial property through possession for at least ten (10) years, service or for the development of the national evidence, conformably with
in good faith and with just title. Under extraordinary acquisitive Article 422 of the Civil Code. The classification of the subject property as
prescription, a person’s uninterrupted adverse possession of alienable and disposable land of the public domain does not change its
patrimonial property for at least thirty (30) years, regardless of good status as property of the public dominion under Article 420(2) of the Civil
faith or just title, ripens into ownership. Code. Thus, it is insusceptible to acquisition by prescription.
2013
3. May a parcel of land established as agricultural in character either Classifications of land according to ownership
because of its use or because its slope is below that of forest lands Land may be classified as either of public dominion or of private ownership.
be registrable under Section 14(2) of the Property Registration It is of public dominion if it:
Decree in relation to the provisions of the Civil Code on acquisitive 1. is intended for public use; or
prescription? 2. belongs to the State, without being for public use, and is intended
4. Are petitioners entitled to the registration of the subject land in their for some public service or for the development of the national
names under Section 14(1) or Section 14(2) of the Property wealth.
Registration Decree or both? Land belonging to the State that is not of such character, or although of such
character but no longer intended for public use or for public service forms
It is clear that the evidence of petitioners is insufficient to establish that part of the patrimonial property of the State. Land that is other than part of
Malabanan has acquired ownership over the subject property under Section the patrimonial property of the State, provinces, cities and municipalities is
48(b) of the Public Land Act. There is no substantive evidence to establish of private ownership if it belongs to a private individual. Pursuant to the
that Malabanan or petitioners as his predecessors-in-interest have been in Regalian Doctrine (Jura Regalia), all lands of the public domain belong to the
possession of the property since 12 June 1945 or earlier. The earliest that State. All lands not appearing to be clearly under private ownership are
petitioners can date back their possession, according to their own presumed to belong to the State. Also, public lands remain part of the
evidence—the Tax Declarations they presented in particular—is to the year inalienable land of the public domain unless the State is shown to have
1948. Thus, they cannot avail themselves of registration under Section 14(1) reclassified or alienated them to private persons.
of the Property Registration Decree.
Classifications of public lands according to alienability
34 Tacder NatRes
1935 Constitution: lands of the public domain were classified into - Disposition of alienable public lands
agricultural, timber and mineral. Section 10, Article XIV of the 1973 Section 11 of the Public Land Act (CA No. 141) provides the manner by which
Constitution: lands of the public domain were classified into - agricultural, alienable and disposable lands of the public domain, i.e., agricultural lands,
industrial or commercial, residential, resettlement, mineral, timber or forest, can be disposed of:
and grazing land, with the reservation that the law might provide other 1. For homestead settlement;
classifications. 1987 Constitution adopted the classification under the 1935 2. By sale;
Constitution into agricultural, forest or timber, and mineral, but added 3. By lease; and
national parks. Under Section 2, Article XII of the 1987 Constitution, only 4. By confirmation of imperfect or incomplete titles;
agricultural lands of the public domain may be alienated; all other natural a) By judicial legalization; or
resources may not be. Alienable and disposable lands of the State fall into b) By administrative legalization (free patent).
two categories:
a) patrimonial lands of the State, or those classified as lands of private
ownership under Article 425 of the Civil Code, without limitation; and The core of the controversy herein lies in the proper interpretation of Section
b) lands of the public domain, or the public lands as provided by the 11(4), in relation to Section 48(b) of the Public Land Act, which expressly
Constitution, but with the limitation that the lands must only be requires possession by a Filipino citizen of the land since June 12, 1945, or
agricultural. earlier. Bearing in mind such limitations under the Public Land Act, the
Consequently, lands classified as forest or timber, mineral, or national parks applicant must satisfy the following requirements in order for his application
are not susceptible of alienation or disposition unless they are reclassified as to come under Section 14(1) of the Property Registration Decree, to wit:
agricultural. A positive act of the Government is necessary to enable such 1. The applicant, by himself or through his predecessor-in-interest, has
reclassification, and the exclusive prerogative to classify public lands under been in possession and occupation of the property subject of the
existing laws is vested in the Executive Department, not in the courts. If, application;
however, public land will be classified as neither agricultural, forest or timber, 2. The possession and occupation must be open, continuous, exclusive,
mineral or national park, or when public land is no longer intended for public and notorious;
service or for the development of the national wealth, thereby effectively 3. The possession and occupation must be under a bona fide claim of
removing the land from the ambit of public dominion, a declaration of such acquisition of ownership;
conversion must be made in the form of a law duly enacted by Congress or 4. The possession and occupation must have taken place since June 12,
by a Presidential proclamation in cases where the President is duly 1945, or earlier; and
authorized by law to that effect. 5. The property subject of the application must be an agricultural land
of the public domain.
35 Tacder NatRes
In sum, these are the rules relative to the disposition of public land or lands may be alienated or disposed through any of the modes of
of the public domain, namely: acquiring ownership under the Civil Code.
1. As a general rule and pursuant to the Regalian Doctrine, all lands of If the mode of acquisition is prescription, whether ordinary or extraordinary,
the public domain belong to the State and are inalienable. Lands proof that the land has been already converted to private ownership prior to
that are not clearly under private ownership are also presumed to the requisite acquisitive prescriptive period is a condition sine qua non in
belong to the State and, therefore, may not be alienated or observance of the law (Article 1113, Civil Code) that property of the State not
disposed; patrimonial in character shall not be the object of prescription. In the case at
2. Exceptions: bar, the petitioners failed to present sufficient evidence to establish that they
a) Agricultural lands of the public domain are rendered and their predecessors-in-interest had been in possession of the land since
alienable and disposable through any of the exclusive June 12, 1945. Without satisfying the requisite character and period of
modes enumerated under Section 11 of the Public Land Act. possession - possession and occupation that is open, continuous, exclusive,
If the mode is judicial confirmation of imperfect title under and notorious since June 12, 1945, or earlier - the land cannot be considered
Section 48(b) of the Public Land Act, the agricultural land ipso jure converted to private property even upon the subsequent
subject of the application needs only to be classified as declaration of it as alienable and disposable. Prescription never began to run
alienable and disposable as of the time of the application, against the State, such that the land has remained ineligible for registration
provided the applicant’s possession and occupation of the under Section 14(1) of the Property Registration Decree. Likewise, the land
land dated back to June 12, 1945, or earlier. Thereby, a continues to be ineligible for land registration under Section 14(2) of the
conclusive presumption that the applicant has performed all Property Registration Decree unless Congress enacts a law or the President
the conditions essential to a government grant arises, and issues a proclamation declaring the land as no longer intended for public
the applicant becomes the owner of the land by virtue of an service or for the development of the national wealth.
imperfect or incomplete title. By legal fiction, the land has
already ceased to be part of the public domain and has Palomo vs CA
become private property. DOCTRINE: 1) Law governing natural resources that forest land cannot be
b) Lands of the public domain subsequently classified or owned by private persons. It is not registrable and possession thereof, no
declared as no longer intended for public use or for the matter how lengthy, cannot convert it into private property, less such lands
development of national wealth are removed from the are reclassified and considered disposable and alienable. 2) Neither do the
sphere of public dominion and are considered converted tax receipts which were presented in evidence prove ownership of the parcels
into patrimonial lands or lands of private ownership that 3 of land inasmuch as the weight of authority is that tax declarations are not
conclusive proof of ownership in land registration cases.
36 Tacder NatRes
FACTS: Whether or not tax receipt can be use as a proof of ownership over
Diego Palomo is the owner of 15 parcels of land covered by Executive the claiming lands.
Order No. 40. On 1916, he ordered the registration of these lands
and donated the same to his heirs, Ignacio and Carmen Palomo two
months before his death in April 1937. RULING:
Averment that the aforesaid OCT were lost during the Japanese There is no question that the lots here forming part of the forest
occupation, Ignacio Palomo filed a petition for reconstitution with zone were not alienable lands of the public domain. The adverse
the Court of First Instance of Albay on May 1970. The Register of possession which may be the basis of a grant of title in confirmation
Deeds of Albay issued Transfer Certificates of Title Nos. 3911, 3912, of imperfect title cases applies only to alienable lands of the public
3913 and 3914 sometime in October 1953. domain. It is in the law governing natural resources that forest land
July 1954 President Ramon Magsaysay issued Proclamation No. 47 cannot be owned by private persons. It is not registerable and
converting the area embraced by Executive Order No. 40 into the possession thereof, no matter how lengthy, cannot convert it into
"Tiwi Hot Spring National Park," under the control, management, private property, unless such lands are reclassified and considered
protection and administration of the defunct Commission of Parks disposable and alienable.
and Wildlife, now a division of the Bureau of Forest Development. Tax receipts which were presented in evidence prove ownership of
Land was never released as alienable and disposable portion of the the parcels of land inasmuch as the weight of authority is that tax
public domain and, therefore, is neither susceptible to disposition declarations are not conclusive proof of ownership in land
under the provisions of the Public Land Law nor registerable under registration cases. As to the forfeiture of improvements introduced
the Land Registration Act. The Palomos, however, continued in by petitioners, the fact that the government failed to oppose the
possession of the property, paid real estate taxes thereon and registration of the lots in question is no justification for petitioners to
introduced improvements by planting rice, bananas, pandan and plead good faith in introducing improvements on the lots.
coconuts.
April 8, 1971, petitioner Carmen de Buenaventura and spouses
Ignacio Palomo and Trinidad Pascual mortgaged the parcels of land
to guarantee a loan of P200,000 from the Bank of the Philippine Bracewell vs CA
Islands. Facts: The controversy involves a total of nine thousand six hundred fifty-
ISSUES: seven (9,657) square meters of land located in Las Piñas, Metro Manila. In
Whether or not forest land which is a public domain may be owned 1908, Maria Cailles, married to James Bracewell, Sr., who acquired the said
by private persons parcels of land from the Dalandan and Jimenez families of Las Piñas; after
37 Tacder NatRes
which corresponding Tax Declarations were issued in the name of Maria b) Whether the tax declarations attached to the complaint do not
Cailles. constitute acquisition of the lands applied for?
Held:
On January 16, 1961, Maria Cailles sold the said parcels of land to her son, the
petitioner, by virtue of a Deed of Sale which was duly annotated and The controversy is simple. On one hand, petitioner asserts his right of title to
registered with the Registry of Deeds of Pasig, Rizal. Tax Declarations were the subject land under Section 48 (b) of Commonwealth Act No. 141, having
thereafter issued in the name of petitioner, canceling the previous Tax by himself and through his predecessors-in-interest been in open,
Declarations issued to Maria Cailles. On September 19, 1963, petitioner filed continuous, exclusive and notorious possession and occupation of the
before the then Court of First Instance of Pasig, Rizal an action for subject parcels of land, under a bona fide claim of acquisition or ownership,
confirmation of imperfect title under Section 48 of Commonwealth Act No. since 1908. On the other hand, it is the respondents' position that since the
141. subject parcels of land were only classified as alienable or disposable on
March 27, 1972, petitioner did not have any title to confirm when he filed his
The Director of Lands, represented by the Solicitor General, opposed application in 1963. Neither was the requisite thirty years possession met.
petitioner's application on the grounds that neither he nor his predecessors-
in-interest possessed sufficient title to the subject land nor have they been in A similar situation in the case of Reyes v. Court of Appeals, where a
open, continuous, exclusive and notorious possession and occupation of the homestead patent issued to the petitioners' predecessor-in-interest was
same for at least thirty (30) years prior to the application, and that the subject cancelled on the ground that at the time it was issued, the subject land was
land is part of the public domain. still part of the public domain.
On May 3, 1989, the lower court issued an Order granting the application of In the said case, this Court ruled as follows —
petitioner. The Solicitor General promptly appealed to respondent Court
which, on June 29, 1992, reversed and set aside the lower court's Order. It Under the Regalian doctrine, all lands of the public domain belong to the
also denied petitioner's Motion for Reconsideration in its Resolution of State, and that the State is the source of any asserted right to ownership in
September 30, 1992. land and charged with the conservation of such patrimony. This same
doctrine also states that all lands not otherwise appearing to be clearly within
Issues: private ownership are presumed to belong to the State (Director of Lands vs.
a) Whether the failure of the petitioner to prosecute his action for an Intermediate Appellate Court, 219 SCRA 340).
unreasonable length of time?
38 Tacder NatRes
Hence, the burden of proof in overcoming the presumption of State acquired ownership and possession of said parcels of land by purchase from
ownership of lands of the public domain is on the person applying for the original owners thereof, whose possession of the same including that of
registration. The applicant must show that the land subject of the application the herein respondents, has always been continuous, open, active, exclusive,
is alienable or disposable. These petitioners failed to do. public, adverse and in the concept of owners for more than 30 years. The
The homestead patent was issued to petitioners' predecessor-in-interest, the Director of Forestry filed an opposition to the above petition but later
subject land belong to the inalienable and undisposable portion of the public withdrew the same upon verification of findings that this portion of the
domain. Thus, any title issued in their name by mistake or oversight is void timberland had already been released from the mass of the public forests.
ab initio because at the time the homestead patent was issued to petitioners, Subsequently, the Acting Prov. Fiscal of Bataan, for and in behalf of the
as successors-in-interest of the original patent applicant, the Director of Director of Lands filed his opposition alleging that the land is still a Public
Lands was not then authorized to dispose of the same because the area was Land and as such cannot be the subject of a land registration proceeding
not yet classified as disposable public land. Consequently, the title issued to under Act 496. The lower court adjudicated in favor or respondent Bernabes,
herein petitioners by the Bureau of Lands is void ab initio. finding that the latter have complied with all the terms and conditions
Neither has petitioner shown proof that the subject Forestry Administrative entitling them to a grant. This decision having become final, the
Order recognizes private or vested rights under which his case may fall. We Commissioner of Land Registration issued the corresponding decrees of
only find on record the Indorsement of the Bureau of Forest Development registration. On the other hand, petitioner DL through the Solicitor Gen. filed
from which no indication of such exemption may be gleaned. a petition for review of the decrees. Afterwards, he filed an Amended Petition
Having found petitioner to have no cause of action for his application for for Review, adding: that respondents executed simulated deeds of sale
confirmation of imperfect title, we see no need to discuss the other errors conveying portions of the subject parcels to third parties for fictitious
raised in this petition. considerations in order to remove the same from the coverage of Sec. 38 of
Act 496, but in truth, buyers are mere dummies of petitioners; hence, not
Republic vs CA and Bernabe purchasers for value. The Court of First Instance denied this petition and on
FACTS: appeal, the CA affirmed the questioned decision. Petitioner’s Motion for
Lot No. 622 of the Mariveles Cadastre was declared public land in a decision Reconsideration having been denied for lack of merit; hence, this petition.
rendered before the last war in Cadastral Case No. 19, LRC Cadastral Record
No. 1097. On July 6, 1965 such lot was segregated from the forest zone and
released and certified by the Bureau of Forestry (BOF) as an agricultural Land ISSUE:
for disposition under the Public Land Act. On April 26, 1967, Respondents WON the lots claimed by respondents could be legally be the subject of a
filed in the CFI of Bataan a petition to reopen Cadastral Case No. 19 to perfect juridical confirmation of Title under Section 48 (b) of Commonwealth Act 141
their rights and register their titles to said lots. They alleged that they as amended by Republic Act 1942.
39 Tacder NatRes
MIAA claims that although the charter provides that the title of the land and
building are with MIAA still the ownership is with the Republic of the
Philippines. MIAA also contends that it is an instrumentality of the
government and as such exempted from real estate tax. That the land and
buildings of MIAA are of public dominion therefore cannot be subjected to
levy and auction sale. On the other hand, the officers of Paranaque City claim
that MIAA is a government owned and controlled corporation therefore not
exempted to real estate tax.
Issues:
Whether or not MIAA is an instrumentality of the government and
not a government owned and controlled corporation and as such
Non-registrable Land exempted from tax.
Whether or not the land and buildings of MIAA are part of the public
dominion and thus cannot be the subject of levy and auction sale.
MIAA vs CA Ruling:
Facts: Manila International Airport Authority (MIAA) is the operator of the Under the Local government code, government owned and controlled
Ninoy International Airport located at Paranaque City. The Officers of corporations are not exempted from real estate tax. MIAA is not a
Paranaque City sent notices to MIAA due to real estate tax delinquency. government owned and controlled corporation, for to become one MIAA
MIAA then settled some of the amount. When MIAA failed to settle the entire should either be a stock or non stock corporation. MIAA is not a stock
amount, the officers of Paranaque city threatened to levy and subject to corporation for its capital is not divided into shares. It is not a non stock
auction the land and buildings of MIAA, which they did. MIAA sought for a corporation since it has no members. MIAA is an instrumentality of the
Temporary Restraining Order from the CA but failed to do so within the 60 government vested with corporate powers and government functions.
days reglementary period, so the petition was dismissed. MIAA then sought
for the TRO with the Supreme Court a day before the public auction, MIAA Under the civil code, property may either be under public dominion or
was granted with the TRO but unfortunately the TRO was received by the private ownership. Those under public dominion are owned by the State and
Paranaque City officers 3 hours after the public auction. are utilized for public use, public service and for the development of national
wealth. The ports included in the public dominion pertain either to seaports
41 Tacder NatRes
or airports. When properties under public dominion cease to be for public constructing its new building on the same site because it is the property of
use and service, they form part of the patrimonial property of the State. the church and it needed the lot for its social projects. However, the
construction of the new municipal building on the same site proceeded.
The court held that the land and buildings of MIAA are part of the public Consequently, petitioner filed a complaint and prayed that it be declared the
dominion. Since the airport is devoted for public use, for the domestic and lawful owner and possessor of Lot 138. Petitioner contends that it does not
international travel and transportation. Even if MIAA charge fees, this is for allegedly lose its possession or ownership over the property if the possession
support of its operation and for regulation and does not change the or use by another of the same is by mere tolerance.
character of the land and buildings of MIAA as part of the public dominion. In its answer, the municipality alleged that said lot was surveyed as property
As part of the public dominion the land and buildings of MIAA are outside of the municipality and that the said municipality alone had possessed the
the commerce of man. To subject them to levy and public auction is contrary said land under the claim of title exclusively for over fifty (50) years, exclusive
to public policy. Unless the President issues a proclamation withdrawing the of all other rights and adverse to all other claimants.
airport land and buildings from public use, these properties remain to be of After due trial, the trial court declared petitioner as the lawful owner and
public dominion and are inalienable. As long as the land and buildings are possessor of Lot 138-B and the Municipality of Buruanga as the lawful owner
for public use the ownership is with the Republic of the Philippines. and possessor of Lots 138-A and 138-C, the said lots being public plaza for
public use. On appeal, the CA affirmed the ownership of petitioner over Lot
138-B but declared Lots 138-A and 138-C as property of public dominion.
Roman Catholic vs Municipality of Buruanga ISSUE: Whether or not petitioner’s open, continuous, exclusive and notorious
FACTS: Petitioner Roman Catholic Bishop of Kalibo was allegedly the lawful possession and occupation of Lot 138 since 1894 and for many decades
owner and possessor of a parcel of residential and commercial land, thereafter vests ipso jure or by operation of law upon it a government grant,
designated as Lot 138. The Roman Catholic Church was built in 1984 in the a vested title, to the subject property.
middle portion of the said lot and has been in existence since then up to the
present. That sometime in 1978, the Municipality of Buruanga constructed its HELD: No. There was no question that petitioner has been in open,
municipal building on the northeastern portion of the Lot 138 after it continuous, exclusive and notorious possession and occupation of Lot 138-B
obtained the permission of the then parish priest of Buruanga on the since 1894 as evidenced by the church structure built thereon but there was
condition that the municipality remove all the improvements it constructed no evidence to show that such possession and occupation extended to Lots
thereon if and when the petitioner needed the said lot. 138-A and 138-C beginning the same period. No single instance of the
When the municipal building was razed by fire in 1989, petitioner, through exercise by the petitioner of proprietary acts or acts of dominion over these
its counsel requested the officials of the municipality to refrain from lots was established. Its unsubstantiated claim that the construction of the
42 Tacder NatRes
municipal building as well as the subsequent improvements thereon was by Issue: WON the lot in question can be subject of registration and
its tolerance does not constitute proof of possession and occupation on the confirmation of title in the name of the private person.
petitioner’s part. Absent the important requisite of open, continuous,
exclusive and notorious possession and occupation thereon since 1894, no Held: The opposition of the Director of Forestry was strengthened by the
government grant or title to Lots 138-A and 138-C had vested upon the appellate court's finding that timber licenses had to be issued to certain
petitioner ipso jure or by operation of law. licensees and even Jose Amunategui himself took the trouble to ask for a
Possession is open when it is patent, visible, apparent, notorious and not license to cut timber within the area. It was only sometime in 1950 that the
clandestine. It is continuous when uninterrupted, unbroken and not property was converted into fishpond but only after a previous warning from
intermittent or occasional; exclusive when the adverse possessor can show the District Forester that the same could not be done because it was classified
exclusive dominion over the land and an appropriation of it to his own use as "public forest”.
and benefit; and notorious when it is so conspicuous that it is generally A forested area classified as forest land of the public domain does not lose
known and talked of by the public or the people in the neighborhood. Use such classification simply because loggers or settlers may have stripped it of
of land is adverse when it is open and notorious. its forest cover. "Forest lands" do not have to be on mountains or in out of
the way places. Swampy areas covered by mangrove trees, nipa palms, and
other trees growing in brackish or sea water may also be classified as forest
Amunategui vs Director of Forestry land. The possession of forest lands, no matter how long, cannot ripen into
Facts: There were two petitions for review on certiorari questioning the private ownership. Therefore, the lot in question never ceased to be classified
decision of the Court of Appeals which declared the disputed property as as forest land of public domain.
forest land, not subject to titling in favor of private persons, Borre and
Amunategui. 1. CIVIL LAW; PUBLIC LAND ACT; FOREST LAND; CLASSIFICATION NOT LOST
The Director of Forestry, through the Provincial Fiscal of Capiz, also filed an EVEN IF IT HAS BEEN STRIPPED OF FOREST COVER; UNLESS RELEASED IN
opposition to the application for registration of title claiming that the land AN OFFICIAL PROCLAMATION AS DISPOSABLE LANDS, RULES ON
was mangrove swamp which was still classified as forest land and part of the CONFIRMATION OF IMPERFECT TITLE DO NOT APPLY. — A forested area
public domain. classified as forest land of the public domain does not lose such classification
Another oppositor, Emeterio Bereber filed his opposition insofar as a portion simply because loggers or settlers may have stripped it of its forest cover.
of Lot No. 885 containing 117,956 square meters was concerned and prayed Parcels of land classified as forest land may actually be covered with grass or
that title to said portion be confirmed and registered in his name. planted to crops by kaingin cultivators or other farmers. "Forest lands" do
not have to be on mountains or in out of the way places. Swampy areas
covered by mangrove trees, nipa palms, and other tress growing in brackish
43 Tacder NatRes
That the possession under claim of ownership of the applicant and his matter of public knowledge that a majority of the lands in the Philippine
predecessors in interest was shown to have been open, notorious, actual, Islands are agricultural lands, that the courts have a right to presume, in the
public and continuous for more than forty-four years past, and that their absence of evidence to the contrary, that in each case the lands are
claim was exclusive of any other right adverse to all other claims; agricultural lands until the contrary is shown. Whatever the land involved in
a particular land registration case is forestry or mineral land must, therefore,
That the applicant now has some one hundred fifty (150) hills of hemp, some be a matter of proof. Its superior value for one purpose or the other is a
eight thousand (8,000) cocoanut trees, a dwelling house, various laborers' question of fact to be settled by the proof in each particular case. The fact
quarters, store-building, large camarin (storehouse of wood, a galvanized that the land is a manglar [mangrove swamp] is not sufficient for the courts
iron and other buildings and improvements on said land. to decide whether it is agricultural, forestry, or mineral land. It may perchance
belong to one or the other of said classes of land. The Government, in the
Issue: WON the land in question cannot be registered? first instance, under the provisions of Act No. 1148, may, by reservation,
decide for itself what portions of public land shall be considered forestry land,
Ruling: The mere fact that a tract of land has trees upon it or has mineral unless private interests have intervened before such reservation is made. In
within it is not of itself sufficient to declare that one is forestry land and the the latter case, whether the land is agricultural, forestry, or mineral, is a
other, mineral land. There must be some proof of the extent and present or question of proof. Until private interests have intervened, the Government,
future value of the forestry and of the minerals. While, as we have just said, by virtue of the terms of said Act (No. 1148), may decide for itself what
many definitions have been given for "agriculture," "forestry," and "mineral" portions of the "public domain" shall be set aside and reserved as forestry or
lands, and that in each case it is a question of fact, we think it is safe to say mineral land.
that in order to be forestry or mineral land the proof must show that it is
more valuable for the forestry or the mineral which it contains than it is for Issue: Whether or not the said land is owned by the government of the
agricultural purposes. (Sec. 7, Act No. 1148.) It is not sufficient to show that Philippines.
there exists some trees upon the land or that it bears some mineral. Land
may be classified as forestry or mineral today, and, by reason of the Ruling: Yes, The court held that the applicant proved and validly supplied
exhaustion of the timber or mineral, be classified as agricultural land the requisites for the registration of the said land into an agricultural land as
tomorrow. And vice-versa, by reason of the rapid growth of timber or the per stated by paragraph 6 of section 54 of Act No. 926. The important
discovery of valuable minerals, lands classified as agricultural today may be prerequisites for registration of land imposed by said section 54, paragraph
differently classified tomorrow. Each case must be decided upon the proof 6, are
in that particular case, having regard for its present or future value for one a) that the land shall be agricultural public land as defined by the Act
or the other purposes. We believe, however, considering the fact that it is a of Congress of July 1, 1902;
45 Tacder NatRes
b) that the petitioner, by himself or his predecessors in interest, shall Compulsory acquisition is the power of the government to acquire private
have been in the open, continuous, exclusive and notorious rights in land without the willing consent of its owner or occupant in order
possession and occupation of the same under a bona fide claim of to benefit the society.
ownership for a period of ten years next preceding the taking effect
of said Act. The government failed to disrupt the said facts presented The said land was inspected by the Municipal and Agrarian Reform Officer,
by the applicant. and upon consensus of the authorities concerned, they decided that the said
land must be placed under compulsory acquisition.
Hence, the court rendered its judgement in favor of the applicant
Petitioners filed an objection on the ground that:
Sta Rosa Dev’t vs CA The area is not appropriate for agricultural purposes.
FACTS: The case is a petition regarding Department of Agrarian Reform The area was rugged in terrain with slopes 18% and above. (which
Adjudication Board’s (DARAB) order of compulsory acquisition of petitioner’s falls under the exception in compulsory acquisition of CARP)
property under the Comprehensive Agrarian Reform Program (CARP). The occupants of the land were illegal settlers or (squatters) who by
no means are entitled to the land as beneficiaries.
Petitioner Sta. Rosa Development Corporation (SRRDC), was the registered
owner of two parcel of land situated at Brgy. Casile, Cabuyao, Laguna. ISSUE:
According to them, these lands are watersheds which provide clean and 1. Whether or not the property in question is covered by CARP despite
potable (drinkable) water to the Canlubang community and that 90 light the fact that the entire property formed part of a watershed area
industries are located in that area. prior to the enactment of R.A No. 6657
They were alleging respondents usurped its rights over their property 2. Whether the petition of land conversion of the parcels of land may
thereby destroying the ecosystem. Since the said land provides water to the be granted?
residents, respondents sought an easement of a right of a way to and from
Barangay Castile, to which, by counterclaim, Sta. Rosa sought ejectment HELD:
against respondents.
Respondents went to the DAR and filed a case for compulsory acquisition of Watershed is one of those enumerated by CARP to be exempt from its
the Sta. Rosa Property under the Comprehensive Agrarian Reform Program. coverage.
Art. 67 of PD 1067 provides that Any watershed or any area of land
adjacent to any surface water or overlying any ground water may
46 Tacder NatRes
be declared by the Department of Natural resources as a protected oppositions to the application. Petitioners (Edna Collado and her co-
area. applicants) allege that they have occupied the Lot since time immemorial.
Watersheds may be defined as an area drained by a river and its Their possession has been open, public, notorious and in the concept of
tributaries and enclosed by a boundary or divide which separates it owners. They paid all real estate taxes and submitted evidence to prove that
from adjacent watersheds. there have been 9 transfers of rights among them and their predecessors-
in-interest. RTC ruled in favor of the petitioners for having presented
We cannot ignore the fact that the disputed parcels of land form a vital part sufficient evidence to establish registrable title over the property.
of an area that need to be protected for watershed purposes. The protection
of watersheds ensures an adequate supply of water for future generations ISSUE:
and the control of flashfloods that not only damage property but cause loss
of lives. Protection of watersheds is an intergenerational responsibility that WON petitioners have registrable title over the Lot. NO.
needs to be answered now.
Petitioners concede that the Lot is inside the literal description of Marikina
Although evidence of petitioners is strong, the Supreme Court opines that Watershed Reservation (MWR). Their main claim over the Lot is that “all
the area must be maintained for watershed purposes for ecological and Presidential proclamations like the proclamation setting aside the MWR are
environmental considerations despite the 88 families who are beneficiaries subject to private rights.” EO 33 (which established the MWR) has a saving
of the CARP. It is important that a larger view of the situation be taken clause that the reservations are “subject to existing private rights, if any there
because of the thousands of residents downstream if the watershed will not be.”
be protected and maintained for its natural purpose.
Under the Regalian Doctrine, all lands not otherwise appearing to be clearly
Despite Supreme Court’s strong opinion of protection of watersheds as an within private ownership are presumed to belong to the State. The Spaniards
intergenerational responsibility, they, however ordered to DARAB to conduct first introduced the doctrine to the Philippines through the Laws of the Indies
a re-evaluation of the case since the said land falls under exception. and the Royal Cedulas, specifically, Law 14, Title 12, Book 4 of the Novisima
Recopilacion de Leyes de las Indias which laid the foundation that "all lands
Collado vs CA that were not acquired from the Government, either by purchase or by grant,
FACTS: Petitioner Edna T. Collado filed with the land registration court an belong to the public domain." Upon the Spanish conquest of the Philippines,
application for registration of a parcel of land (“Lot”), situated in Antipolo ownership of all "lands, territories and possessions" in the Philippines passed
Rizal. Attached to the application was a technical description, stating “this to the Spanish Crown.
survey is inside IN-12 Mariquina Watershed.” The Solicitor General filed
47 Tacder NatRes
The Laws of the Indies were followed by the Ley Hipotecaria or the Mortgage whose title still remained in the government and are thrown open to private
Law of 1893. The Spanish Mortgage Law provided for the systematic appropriation and settlement, and excluded the patrimonial property of the
registration of titles and deeds as well as possessory claims. The Royal Decree government and the friar lands."
of 1894 or the "Maura Law" partly amended the Mortgage Law as well as the
Law of the Indies. The Maura Law was the last Spanish land law promulgated Thus, it is plain error for petitioners to argue that under the Philippine Bill of
in the Philippines. It required the "adjustment" or registration of all 1902 and Public Land Act No. 926, mere possession by private individuals of
agricultural lands, otherwise the lands would revert to the state. lands creates the legal presumption that the lands are alienable and
disposable.
Four years later, Spain ceded to the government of the United States all
rights, interests and claims over the national territory of the Philippine Islands Both the 1935 and 1973 Constitutions prohibited the alienation of all natural
through the Treaty of Paris of December 10, 1898. In 1903, the United States resources except agricultural lands of the public domain. The 1987
colonial government, through the Philippine Commission, passed Act No. Constitution readopted this policy. Indeed, all lands of the public domain as
926, the first Public Land Act, which was described as follows: well as all natural resources enumerated in the Philippine Constitution belong
to the State.
"Act No. 926, the first Public Land Act, was passed in pursuance of the
provisions of the Philippine Bill of 1902. The law governed the disposition of Watershed Reservation is a Natural Resource: The term "natural resource"
lands of the public domain. It prescribed rules and regulations for the includes "not only timber, gas, oil coal, minerals, lakes, and submerged lands,
homesteading, selling and leasing of portions of the public domain of the but also, features which supply a human need and contribute to the health,
Philippine Islands, and prescribed the terms and conditions to enable welfare, and benefit of a community, and are essential to the well-being
persons to perfect their titles to public lands in the Islands. It also provided thereof and proper enjoyment of property devoted to park and recreational
for the "issuance of patents to certain native settlers upon public lands," for purposes."
the establishment of town sites and sale of lots therein, for the completion
of imperfect titles, and for the cancellation or confirmation of Spanish Did petitioners acquire private rights over the parcel of land prior to
concessions and grants in the Islands." In short, the Public Land Act operated the issuance of EO 33? NO.
on the assumption that title to public lands in the Name: I. Concept of Jura
Regalia Natural Resources First Set_ Philippine Islands remained in the An applicant must overcome the presumption that the land he is applying
government; and that the government’s title to public land sprung from the for is part of the public domain and that he has an interest to warrant
Treaty of Paris and other subsequent treaties between Spain and the United registration in his name arising from an imperfect title (may have been
States. The term "public land" referred to all lands of the public domain derived from old Spanish grants or titles). In the case at bar, petitioners were
48 Tacder NatRes
unable to acquire a valid and enforceable right or title because of the failure Appeals. The Director of Forestry then came to this Court in a petition for
to complete the required period of possession (at least 30 years). review on certiorari claiming that the land in dispute was forestal in nature
and not subject to private appropriation. He asks that the registration be
Assuming that the Lot was alienable and disposable land prior to the issuance reversed. It is undisputed by the parties that the land in dispute is a mangrove
of EO 33 in 1904, EO 33 reserved the Lot as a watershed. Since then, the Lot land HOWEVER the legal nature of mangrove swamps or manglares are still
became non-disposable and inalienable public land. At the time petitioners in contention. Director of Forestry claims that it is forestall and is not
filed their application on April 25, 1985, the Lot has been reserved as a disposable. On the other hand, Private respondents insists that it is alienable
watershed under EO 33 for 81 years prior to the filing of petitioners’ as agricultural land.
application.
ISSUES: Are mangrove swamps classified as public forest lands?
NOTES: 2. Republic vs Dela Rosa (160 SCRA 228, G.R. No. L-43938, April 15,
1988) FACTS: The case is about a parcel of land whose ownership is disputed RULING: YES. Part of our public forest lands, they are not alienable under the
by four parties: the dela Rosas, Benguet Consolidated Inc (BCI), Atok Corp, Constitution or are they considered public agricultural lands; they may be
and the Bureau of Forestry Development (BFD). In 1965, Jose de la Rosa on acquired under private ownership.
his and on his three children’s behalf, applied to register a parcel of land
divided into 9 lots in Benguet. According to the children, they acquired the Mangrove swamps or manglares should be understood as comprised within
land by virtue of prescription. As evidence they produced tax declarations the public forests of the Philippines as defined in the aforecited Section 1820
and realty tax receipts. of the Administrative Code of 1917. The legislature having so determined, we
have no authority to ignore or modify its decision, and in effect veto it, in the
exercise of our own discretion. The statutory definition remains unchanged
to date and, no less noteworthy, is accepted and invoked by the executive
Director of Forestry vs Villareal department. More importantly, the said provision has not been challenged
FACTS: The said land consists of 178,113 square meters of mangrove swamps as arbitrary or unrealistic or unconstitutional assuming the requisite
located in the municipality of Sapian, Capiz. Ruperto Villareal applied for its conditions, to justify our judicial intervention and scrutiny. The law is thus
registration on January 25, 1949, alleging that he and his predecessors-in- presumed valid and so must be respected. We repeat our statement in
interest had been in possession of the land for more than forty years. He was the Amunategui case that the classification of mangrove swamps as forest
opposed by several persons, including the petitioner on behalf of the lands is descriptive of its legal nature or status and does not have to be
Republic of the Philippines. After trial, the application was approved by the descriptive of what the land actually looks like. That determination having
Court of First Instance of Capiz. The decision was affirmed by the Court of
49 Tacder NatRes
been made and no cogent argument having been raised to annul it, we have The said Declaration of Location of mineral claim was duly recorded in the
no duty as judges but to apply it. Office of the Mining Recorder sometime on January 2, 1931. Fredia mineral
claim, together with other mineral claims, was sold by A.I. Reynolds to Big
It follows from all this that the land under contention being admittedly a part Wedge Mining Company, the earlier corporate name of Atok Big Wedge
of the mangrove swamps of Sapian, and for which a minor forest license had Mining Company, Inc. (Atok for short; herein petitioner) in a Deed of Sale
in fact been issued by the Bureau of Forestry from 1920 to 1950, it must be executed on November 2, 1931. Since then petitioner Atok has been in
considered forest land. It could therefore not be the subject of the adverse continuous and exclusive ownership and possession of said claim up to the
possession and consequent ownership claimed by the private respondent in present.
support of his application for registration. To be so, it had first to be released
as forest land and reclassified as agricultural land pursuant to the certification Atok has paid the realty taxes and occupation fees for the Fredia mineral
the Director of Forestry may issue under Section 1827 of the Revised claim. The Fredia mineral claim together with other mineral claims owned by
Administrative Code. Atok has been declared under Tax Declaration No. 9535 and that in view of
Presidential Decree No. 1214 an application for lease was filed by Atok
The Respondent even showed, a survey of the land and its tax declaration to covering the Fredia mineral claim.
support its claim, however the court held that the same is insufficient
especially now that the land is a forest land. On the other hand, private respondent Liwan Consi has a lot below the land
of a certain Mr. Acay at Tuding Slide, Itogon, Benguet. He constructed a
WHEREFORE, the decision of the Court of Appeals is SET ASIDE and the house thereon sometime in 1964. The lot is covered by Tax Declaration No.
application for registration of title of private respondent is DISMISSED, with 9462. When he first constructed his house below the lot of Mr. Acay he was
cost against him. This decision is immediately executory. told that it was not necessary for him to obtain a building permit as it was
only a nipa hut. And no one prohibited him from entering the land so he was
constructing a house thereon. It was only in January 1984 when private
Atok-Big Wedge Mining Corp vs CA respondent Consi repaired the said house that people came to take pictures
FACTS: Fredia Mineral claim of about nine (9) hectares situated in Tuding, and told him that the lot belongs to Atok. Private respondent Consi has been
Itogon, Benguet, was located sometime between December 25, 1930 and paying taxes on said land which his father before him had occupied .
December 31, 1930, a period of six (6) days, by A.I. Reynolds in accordance
with the provisions of the Act of Congress of July 1, 1902, better known as the On January 1984, the security guards of Atok informed Feliciano Reyes,
Philippine Bill of 1902, in a so-called Declaration of Location. Security Officer of Atok, that a construction was being undertaken at the area
of the Fredia mineral claim by private respondent Liwan Consi. Feliciano
50 Tacder NatRes
ISSUE: It is, therefore, evident that Benguet and Atok have exclusive rights to the
Whether or not an individual's long term occupation of land of the public property in question by virtue of their respective mining claims which they
domain vests him with such rights over the same as to defeat the rights of validly acquired before the Constitution of 1935 prohibited the alienation of
the owner of that claim. all lands of the public domain except agricultural lands, subject to vested
51 Tacder NatRes
rights existing at the time of its adoption. The land was not and could not It is therefore clear that from the legal viewpoint it was really petitioner who
have been transferred to the private respondents by virtue of acquisitive was in actual physical possession of the property. Having been deprived of
prescription, nor could its use be shared simultaneously by them and the this possession by the private respondent, petitioner has every right to sue
mining companies for agricultural and mineral purposes (Ibid). for ejectment.
On the matter of possession, private respondent contends that his With this ruling enunciated by the Court, it can further be declared and held
predecessor-in-interest has been in possession of said lot even before the that petitioner Atok has the exclusive right to the property in question.
war and has in fact cultivated the same. Since the subject lot is mineral land,
private respondent's possession of the subject lot no matter how long did
not confer upon him possessory rights over the same. Republic vs Southside Homeowners
FACTS: The subject matter of these proceedings for declaration of nullity of
Furthermore, Article 538 of the New Civil Code provides: title are parcels of land with a total area of 39.99 hectares, more or less,
known as the JUSMAG housing area in Fort Bonifacio where, military officers,
Art. 538. Possession as a fact cannot be recognized at the same time both in the active and retired services, and their respective families, have
in two different personalities except in the cases of co-possession. Should a been occupying housing units and facilities originally constructed by the AFP.
question arise regarding the fact of possession, the present possessor shall
be preferred; if there are two possessors, the one longer in possession; if the Private respondent SHAI is a non-stock corporation organized mostly by
dates of the possession are the same, the one who presents a title; and if all wives of AFP military officers. Records show that SHAI was able to secure
these conditions are equal, the thing shall be placed in judicial deposit from the Registry of Deeds of the Province of Rizal a title – Transfer Certificate
pending determination of its possession or ownership through proper of Title in its name to the bulk of, if not the entire, JUSMAG area.
proceedings. The Rizal Registry issued TCT No. 15084 on October 30, 1991on the basis of
a notarized Deed of Sale purportedly executed on the same date by then
Since 1931 up to the present, petitioner ATOK has been in continuous and Director Abelardo G. Palad, Jr. of the Lands Management Bureau (LMB) in
exclusive possession of the Fredia mineral claim while private respondent's favor of SHAI.The total purchase price as written in the conveying deed was
possession started only sometime in 1964 when he constructed a house P11,997,660.00 or P30.00 per square meter
thereon. Clearly, ATOK has superior possessory rights than private It appears that in the process of the investigation conducted by the
respondent, Liwan Consi, the former being "the one longer in possession." Department of Justice on reported land scams at the FBMR, a copy of the
aforesaid October 30, 1991deed of sale surfaced and eventually referred to
the National Bureau of Investigation (NBI) for examination. The results of the
52 Tacder NatRes
examination undertaken by NBI Document Examiner Eliodoro Constantino For its part, then defendant SHAI presented an opposing expert witness in
reveals that the puported signatures in the document are forgeries. the person of Police Inspector Redencion Caimbon who testified that Palad’s
signature in Exhibit “A” is genuine. Mrs. Virginia Santos, then SHAI president,
On October 16, 1993, then President Fidel V.Ramos issued Memorandum likewise testified, saying that applications to purchase were signed and then
Order No. 173 directing the Office of the Solicitor General (OSG) to institute filed with the LMB by one Engr. Eugenia Balis, followed by the payment in
action towards the cancellation of TCT No. 15084 and the title acquired by full of the contract price.
the Navy Officer’s Village Association (NOVA) over a bigger parcel within the
reservation. A month later, the OSG, in behalf of the petitioner Republic, filed Eventually, in a decision dated October 7, 1997, the trial court rendered
with the RTC of Pasig City the corresponding nullification and cancellation of judgment dismissing the Republic’s complaint as it considered the parcels
title suit against the private respondent SHAI, purported signature thereon covered by the deed in question as no longer part of the FBMR. Therefrom,
of Palad is a forgery; b) there are no records with the LMB of (i) the the Republic went on appeal to the CA which affirmed in toto that of the trial
application to purchase and (ii) the alleged payment of the purchase price; court.
and c) the property in question is inalienable, being part of a military
reservation established under Proclamation No. 423. Hence, this petition of the Republic.
On pre-trial the Republic, as plaintiff therein, marked (and later offered in
evidence)the Deed of Sale dated October 30, 1991 as its Exhibit "A,"and TCT ISSUE: Was the JUSMAG area, during the period material, alienable or
No. 15084 as Exhibit "B."Respondent, then defendant SHAI adopted Exhibits inalienable, as the case may be, and, therefore, can or cannot be subject of
"A" and “B” as its Exhibits "1" and “2,” respectively. a lawful private conveyance?
During the trial, the Republic presented as expert witness NBI Document RULING: Petitioner Republic, correctly asserts the inalienable character of the
Examiner Eliodoro Constantino who testified on NBI QDR No. 815-1093 and JUSMAG area, the same having not effectively been separated from the
asserted that the signature of Palad in Exhibit “A” is a forgery. For his part, military reservation and declared as alienable and disposable.
Palad dismissed as forged his signature appearing in the same document
and denied ever signing the same, let alone in front of a notary public The President, upon the recommendation of the Secretary of Environment
holding office outside of the LMB premises. Pressing the point, Palad stated and Natural Resources, may designate by proclamation any tract or tracts of
that he could not have had signed the conveying deed involving as it did a land of the public domain as reservations for the use of the Republic or any
reservation area which, apart from its being outside of the LMB’s jurisdiction, of its branches, or for quasi-public uses or purposes. Such tract or tracts of
is inalienable in the first place. land thus reserved shall be non-alienable and shall not be subject to sale or
other disposition until again declared alienable. Consistent with the
53 Tacder NatRes
foregoing postulates, jurisprudence teaches that a military reservation, like The October 30, 1991 Deed of Sale purportedly executed by Palad, assuming
the FBMR, or a part thereof is not open to private appropriation or its authenticity, could not plausibly be the requisite classifying medium
disposition and, therefore, not registrable, unless it is in the meantime converting the JUSMAG area into a disposable parcel. And private
reclassified and declared as disposable and alienable public land. And until a respondent SHAI’s unyielding stance that would have the Republic in
given parcel of land is released from its classification as part of the military estoppel to question the transfer to it by the LMB Director of the JUSMAG
reservation zone and reclassified by law or by presidential proclamation as area is unavailing. It should have realized that the Republic is not usually
disposable and alienable, its status as part of a military reservation remains, estopped by the mistake or error on the part of its officials or agents.
even if incidentally it is devoted for a purpose other than as a military camp
or for defense. The same is true in this case. Since the parcels of land in question allegedly sold to the private respondent
are, or at least at the time of the supposed transaction were, still part of the
There is no doubt that the JUSMAG area subject of the questioned October FBMR, the purported sale is necessarily void ab initio.
30, 1991sale formed part of the FBMR as originally established under
Proclamation No. 423. And while private respondent SHAI would Moreover, Article XII, Section 3[of the 1987 Constitution forbids private
categorically say that the petitioner Republic had not presented evidence corporations from acquiring any kind of alienable land of the public domain,
that “subject land is within military reservation,” and even dared to state that except through lease for a limited period.
the JUSMAG area is the private property of the government and
therefore removed from the concept of public domain per se its own The interplay of compelling circumstances and inferences deducible from the
evidence themselves belie its posture as their evidence both the TCT and the case, also cast doubt on the authenticity of such deed, if not support a
Deed of Sale technically described the property as situated in Jusmag area conclusion that the deed is spurious.
located at Fort Bonifacio which is now renamed Fort Mckinley a declared a
military reservation. 1. Palad categorically declared that his said signature on the deed is a
forgery. The NBI signature expert corroborated Palad’s allegation on
The Republic has, since the filing of its underlying complaint, invoked forgery.Respondent SHAI’s expert witness from the PNP, however,
Proclamation No. 423. In the process, it has invariably invited attention to the disputes the NBI’s findings. In net effect, both experts from the NBI
proclamation’s specific area coverage to prove the nullity of TCT No. 15084, and the PNP cancel each other out.
inasmuch as the title embraced a reserved area considered inalienable, and 2. Palad signed the supposed deed of sale in Manila, possibly at the
hence, beyond the commerce of man. LMB office at Plaza Cervantes, Binondo. Even if he acted in an official
capacity, Palad nonetheless proceeded on the same day to Pasig
City to appear before the notarizing officer. The deed was then
54 Tacder NatRes
brought to the Rizal Registry and there stamped “Received” by the A contract of sale is void where the price, which appears in the document
entry clerk. That same afternoon, or at 3:14 p.m. of October 30, as paid has, in fact, never been paid.
1991to be precise, TCT No. 15084 was issued. In other words, the
whole conveyance and registration process was done in less than a 5. The purchase price was, according to the witnesses for SHAI, paid in
day. The very unusual dispatch is quite surprising. Stranger still is full in cash to the cashier of the LMB the corresponding amount
why a bureau head, while in the exercise of his functions as the apparently coming in a mix of P500 and P100 denominations. Albeit
bureau’s authorized contracting officer, has to repair to another city plausible, SHAI’s witnesses’ account taxes credulity to the limit.
just to have a deed notarized.
3. There is absolutely no record of the requisite public land application TCT No. 15084 of the Registry of Deeds of Rizal issued on the basis of such
to purchase required under Section 89 of the Public Land Act. There Deed are declared void and cancelled
is also no record of the deed of sale and of documents usually
accompanying an application to purchase, inclusive of the
investigation report and the property valuation. The Certification Republic vs Alagad
under the seal of the LMB bearing date November 24, 1994 and Facts: On Oct. 11, 1951, Melitona, Carmen (with spouse Espiridion Kolimlim),
issued/signed by Alberto Recalde, OIC, Records Management Justo, Carlos, Librada (with spouse Emerson Abano), Demetrio, and Antonio
Division of the LMB pursuant to a subpoena issued by the trial court Alagad filed an application for registration of their title over a parcel of land
attest to this fact of absence of records. Atty. Alice B. Dayrit, then situated at Linga, Pila, Laguna, with an area of 8.1263 hectares, which was
Chief, Land Utilization and Disposition Division, LMB, testified having amended after the land was divided into two parcels, namely, Lot 1 withan
personally looked at the bureau record book, but found no entry area of 5.2476 hectares and Lot 2 with an area of 2.8421 hectares. The
pertaining to SHAI. Republic opposed the application on the stereo-typedground that applicants
4. In its Answer as defendant a quo, respondent SHAI states that the and their predecessors have not been in possession of the land openly,
“deed of sale specifically meritorious Official Receipt No. 6030203 as continuously, publicly and adversely under a bona fide claim of ownership
evidence of full payment of the agreed purchase price” An official since July 26, 1894 and the land has not ceased to be a part of the public
receipt (O.R.) is doubtless the best evidence to prove payment. While domain. It appears that barrio folk also opposed the application. On 16
it kept referring to O.R. No. 6030203 as its evidence of the required January 1956, by virtue of a final judgment in said case, supplemented
payment, it failed to present and offer the receipt in evidence. We byorders issued on March 21, 1956 and Aug. 13, 1956, the Alagads were
can thus validly presume that no such OR exists or, if it does, that its declared owners of Lot 1 and the remaining portion, or Lot2, was declared
presentation would be adverse to SHAI. public land. In August 1966, the Alagads filed before the Municipal Court of
Pila, Laguna an action to evict the barrio folk occupying portions of Lot 1. On
55 Tacder NatRes
8 August 1968, judgment was rendered in the eviction case ordering the of the most rigorous scrutiny before private claims to portions thereof are
barrio folktherein to return possession of the premises to the Alagads. The judicially accorded recognition. Such primordial consideration, not the
barrio folk did not appeal. The Republic filed a petition for “annulment of title apparent carelessness, much less the acquiescence of public officials, is the
and reversion, insofar as the 1.42 hectare northwestern portion on end of controlling norm.
Lot1 is concerned, contending that such is foreshore land, and that the
Alagads could not have had an imperfect title to it as it was the barrio folk Ramos v. Central Bank, and Nilo v. Romero not applicable to the present case
who filled up the land to elevate the land to its present condition. The Court, The cases of Ramos v. Central Bank of the Philippines and Nilo v. Romero,
issued a writ of preliminary injunction enjoining the Provincial Sheriff of are not applicable. In Ramos, the Court applied estoppel upon finding of bad
Laguna or his deputies from enforcing a writ of execution, and the Alagads faith on the part of the State (the Central Bank) in deliberately reneging on
from selling, mortgaging, disposing or otherwise entering into any its promises. In Nilo, the Court denied efforts to impugn the jurisdiction of
transaction affecting the area. The case was set for pre-trial on July6, 1971, to the court on the ground that the defendant had been “erroneously”
which the attorney representing the Republic did not appear. On July 16, 1971, represented in the complaint by the City Attorney when it should have been
the court dismissed the complaint. The Republic filed a motion for the City Mayor, on a holding that the City Attorney, in any event, could have
reconsideration, was set for hearing, and finally denied by the court. Appeal ably defended the City (Davao City). In both cases, it is seen that the acts that
was made to the Court of Appeals, which sustained the trial court for failure gave rise to estoppel were voluntary and intentional in character, in which
to show in the record on appeal that the appeal was perfected on time. cases, it could not be said that the Government had been prejudiced by some
Hence, the appeal. The Supreme Court reversed the decision of the lower negligent act or omission.
courts, and reinstated the Republic’s complaint and thus remanded the case
to the trial court for further proceedings Res judicata is not an impediment to reversion of property;
Republic v. CA, requisites for a prior judgment to become a bar Res judicata
is not an impediment to reversion of property. In Republic v. Court of
State cannot be bound by or estopped from the mistakes or negligent acts Appeals, the Court stated that a certificate of title may be ordered cancelled
of its officials or agents (Republic v. Animus, et al.), and the cancellation may be pursued through an
The State cannot be bound by, or estopped from, the mistakes or negligent ordinary action therefor. This action cannot be barred by the prior judgment
acts of its official or agents, much more, non-suited as a result thereof. This of the land registration court, since the said court had no jurisdiction over
is so because the state as a persona in law is the judicial entity, which is the the subject matter. And if there was no such jurisdiction, then the principle
source of any asserted right to ownership in land under the basic doctrine of res judicata does not apply. For it is a well-settled rule that for a prior
embodied in the 1935 Constitution as well as the present charter. It is charged judgment to constitute a bar to a subsequent case, the following requisites
moreover with the conservation of such patrimony. There is need therefore must concur;
56 Tacder NatRes
Patrimonial property and property of public dominion It is also ordained in Article 44 of the Spanish Law of Waters of 3 August 1866
“All other property of the State which is not of the character mentioned in that “natural ponds and lakes existing upon public lands and fed by public
article [420], is patrimonial property,” meaning to say, property “open to waters, belong to the public domain. Lakes, ponds, and pools existing upon
disposition” by the Government, or otherwise, property pertaining to the the lands of private individuals, or the State or provinces, belong to the
national domain, or public lands. Property of the public dominion, on the respective owners of such lands, and those situated upon lands of communal
other hand, refers to things held by the State by regalian right. They are use belong to their respective ‘pueblos.’”
things res publicae in nature and hence, incapable of private appropriation.
Thus, under the present Constitution, “[w]ith the exception of agricultural Laguna de Bay is a lake (Colegio de San Jose case);
lands, all other natural resources shall not be alienated.”
57 Tacder NatRes
Highest Ordinary Depth Laguna de Bay has long been recognized as a lake. Court cannot make a ruling because it is not a trier of facts, and it is in
Laguna de Bay is a body of water formed in depressions of the earth; it possession of no evidence to assist it in arriving at a conclusive disposition.
contains fresh water coming from rivers and brooks or springs, and is The Court thus remanded the case to the court a quo to determine whether
connected with Manila Bay by the Pasig River. Inasmuch as Laguna de Bay is or not the property subject of controversy is foreshore.
a lake, the Court must resort to the legal provisions governing the ownership
and use of lakes and their beds and shores, in order to determine the
character and ownership of the parcels of land in question. The recourse to
legal provisions is necessary, for under Article 74 of the Law of Waters, “the SIAN Enterprise vs FF Cruz
natural bed or basin of lakes is the ground covered by their waters when at That the foreshore area had been reclaimed does not remove it from its
their highest ordinary depth” and in which case, it forms part of the national classification of foreshore area subject to the preferential right to lease of the
dominion. When Laguna de Bay’s waters are at their highest ordinary depth littoral owner.
has been defined as the highest depth of the waters of Laguna de Bay during FACTS: Western Visayas Industrial Corporation (WESVICO) filed a foreshore
the dry season, such depth being the “regular, common, natural, which lease application over the foreshore land adjacent to certain
occurs always or most of the time during the year. Otherwise, where the rise lots registered in its name. It eventually withdrew the application and filed a
in water level is due to the “extraordinary” action of nature, rainfall for petition for registration over the same foreshore land with the then Court of
instance, the portions inundated thereby are not considered part of the bed First Instance of Iloilo. The case was, however, archived as WESVICO‘s
or basin of the body of water in question. It cannot therefore be said to be representative could no longer be contacted, and later on, WESVICO has
foreshore land but land outside of the public dominion, and land capable of ceased operations.
registration as private property.
F.F. Cruz & Co. (F.F. Cruz) filed with the Bureau of Lands, Iloilo City a foreshore
Foreshore land lease application over a foreshore land, a portion of which is adjacent to the
A foreshore land has been defined as “that part of (the land) which is between lot previously occupied by WESVICO. Sian Enterprises Inc. (SIAIN) purchased
high and low water and left dry by the flux and reflux of the tides,” or “The the properties previously owned by WESVICO from the Development Bank
strip of land that lies between the high and low water marks and that is of the Philippines. It subsequently filed a foreshore lease application over the
alternatively wet and dry according to the flow of the tide.” If the foreshore land adjacent to the properties it bought from DBP.
submergence, however, of the land is due to precipitation, it does not
become foreshore, despite its proximity to the waters. 9. Court not a trier of Upon learning that 130 linear meters of the foreshore land subject of F.F.
facts; not enough evidence to arrive a conclusive disposition; Remand The Cruz’s foreshore lease application overlapped that covered by its foreshore
case has to be decided alongside the above principles and regretfully, the lease application, SIAIN filed a protest 8 alleging that it being the owner of
58 Tacder NatRes
the property adjoining the overlapping area, it should be given preference Contrary to the ruling of the Office of the President, as affirmed by
in its lease. the appellatecourt, littoral owner WESVICO cannot be considered to have
waived or abandoned its preferential right to lease the disputed area when
F.F. Cruz, argued that SIAIN must not be given preferential right since the it subsequently filed an application for registration thereover. For being a
area in dispute is classified as ―reclaimed‖ and that the ownership was not part of the public domain, ownership of the area could not be acquired by
by means of accretion. This argument has been sustained by the Land WESVICO. Its preferential right remained, however. Its move to have
Management Bureau. the contested land titled in its name, albeit a faux pas, in fact more than
proves its interest to utilize it.
Upon appeal to the DENR Secretary, SIAIN was upheld, declaring that there
was no basis to declare the area as ―reclaimed‖ . F.F. Cruz however appealed As correctly argued by SIAIN, were WESVICO‘s petition for registration
to the Office of the President which overturned the decision of the DENR which, as stated earlier, was archived by the trial court, pursued but
Secretary and found that the area is reclaimed. On appeal, the Court eventually denied, WESVICO would not have been barred from filing anew a
of Appeals affirmed the decision. Hence, the present petition. SIAIN foreshore lease application. Parenthetically, the petition for registration of
contends that the evidence overwhelmingly proves that the disputed area is WESVICO was archived not on account of lack of interest but because it
foreshore land and not reclaimed land which thus entitles it preferential ceased operations due to financial reasons.
rights over the
HELD: That the foreshore area had been reclaimed does not remove it from
its classification of foreshore area subject to the preferential right to lease of
the littoral owner.
It bears noting that it was not the reclamation
that brought the disputed foreshore area into existence. Such foreshore area
existed even before F.F. Cruz undertook its reclamation. It was ―formed by
accretions or alluvial deposits due to the action of the sea.‖ Following
Santulan, the littoral owner has preferential right to lease the same.
59 Tacder NatRes
Cadastral Registration Proceedings there is no plaintiff and there is no defendant. In another sense, the
a) Section 53 CA 141 Government is the plaintiff and all the- claimants are defendants.
b) Section 35-38 PD 1529
of these lands to him had not been formally accepted according to Article A careful examination of the decision of this Court in the previous case (37
633 of the Civil Code. Upon appeal to this Court, the judgment of the trial Phil., 865) convinces us that there is no res judicata. We merely held that
court was affirmed on the second ground aforementioned Abellera had not acquired title to the hacienda until the execution of the
(Abellera vs. Balanag G.R. No. 11970, promulgated March 22, 1918, and deed of acceptance and the notification thereof, and we clearly refused to
reported in 37 Phil. 865). prevent Abellera from instituting a new action based upon his assertion that
It appears in that decision of this Court that after the perfection of the appeal, he had acquired title to the estate since the dismissal of his original action.
Abellera executed a public document formally accepting the donation of the The other ground for the motion for dismissal, prescription, is not involved
land, and presented and deed of acceptance together with proofs of in the present proceedings.
notification of acceptance to the donor, as ground for new trial. This Court The next question is: Did the cadastral court, on the ground of res judicata,
held that this was not newly-discovered evidence, and that Abellera had not have any power to entertain the motion to dismiss Abellera's claim and bar
acquired title to the hacienda until the execution of the deed of acceptance him from presenting evidence to prove his ownership of these lots?
and the notification thereof to donor. This Court added: Rule 132 of the Rules of Court provides:
So that whether rights he may have to institute and maintain a new action of These rules shall not apply to land registration, cadastral and election cases,
ejectment in reliance upon his claim that he has acquired title to naturalization and insolvency proceedings, and other cases not herein
the hacienda, since the date of the dismissal of this action, it is clear that the provided for, except by analogy or in a suppletory character and whenever
present action was properly dismissed on the ground of failure of proof of practicable and convenient.
title in the plaintiff at the time when the action was instituted and later when The Rules of Court may be applied in cadastral cases when two conditions
judgment of dismissal was entered by the trial court. are present: (1) analogy or need to supplement the cadastral law, and (2)
In July of 1918, or four months after the above-mentioned decision of this practicability and convenience.
Court, petitioner herein brought another action for recovery of the land If the nature and objective of the cadastral scheme are kept in view, a motion
against the same defendants in the previous case. The second suit was later to dismiss in a cadastral case on the ground of prior judgment would seem
dismissed by the Court of First Instance and transferred to cadastral case No. to be out of place. The Government initiates a cadastral case, compelling all
5 which included the hacienda in question that had in the meantime been claimants in a municipality to litigate against one another regarding their
subdivided into lots. When the cadastral case came up before the Hon. respective claims of ownership. By this plan, all the private lands in a town
Meynardo M. Farol at Aringay, La Union, in July 1941, Fabian B.S. Abellera are registered in one single collective proceeding. Thus, the piece-meal and
appeared as claimant while Narciso de Guzman and others appeared as isolated registration of lands, so inadequate in more ways than one, is
adverse claimant. The latter through counsel moved that Abellera's claim avoided. The principal aim is to settle as much as possible all disputes over
over the lots concerned be dismissed on the grounds of res judicata and land and to remove all clouds over land titles, as far a practicable, in a
prescription. community. To attain this purpose, the cadastral court should allow all
61 Tacder NatRes
claimants ample freedom to ventilate whatever right they may assert over
real estate, permitting them, in keeping with the law of evidence, to offer
proofs in support of their allegations. To countenance the contrary opinion, Prohibited Alienations and Transfer of Private Lands
a) Sections 118-124 of CA 141
by suppressing the presentation of evidence in support of claims, would but b) Section 7 Art XII, 1987 Constitution
serve to perpetuate conflicts over land, for such stifled affirmations of
ownership will fester like wounds unskillfully treated. No sufficient leeway
having been give all claimants to demonstrate the strength and consistently
Sections 118-124 of CA 141
of their alleged rights, the stability of decrees of title is jeopardized.
In Haw Pia vs. Roman A. Cruz (G.R. No. 48506), we declared that the Court
of First Instance in a cadastral proceeding cannot appoint a receiver because Gayapanao vs IAC
its jurisdiction is special and limited. We declined in that case to apply the FACTS: This is a petition for review on certiorari filed by Severino Gayapanao
new Rules of Court by analogy. and his siblings questioning the decision of the IAC in upholding the validity
We are, therefore, of the opinion that while in a cadastral case res judicata is of the sale of the land between their father Constantino and their sister. The
available to a claimant in order to defeat the alleged rights of another 2 hectare land, subject of this case is part of 10 hectare homestead land
claimant, nevertheless prior judgment can not set up in a motion to dismiss. registered in the name of Constantino Gayapanao under OCT. The final order
The order appealed from is hereby reversed. Petitioner herein shall in the of the Director of Lands for the issuance of patent was issued on December
cadastral proceedings be allowed to present evidence to prove his claim over 10, 1937. On November 15, 1938, Constantino Gayapanao executed a private
the lots in question. With costs against the adverse claimants who are deed entitled kasulatan ng bilihan in favor of Simeona Gayapanao and his
respondents herein. So ordered. husband.
Yulo, C.J., Moran and Ozaeta, JJ. concur.
RTC’s decision: The contract of sale between Simeona Gayapanao and her
father is null and void for having been executed with the 5 year prohibitory
period provided under Section 118 of the Public Land Law
CA’s decision: It reversed the decision of the RTC and uphold the validity of
the sale.
RULING: No, the provision of law which prohibits the sale or encumbrance Ramos and Leon de Guzman 19 hectares of the homestead land (Exhibit
of the homestead within 5 years after the grant is mandatory. A). The vendees took possession of the part sold to them. The deed of sale
was not submitted to the Secretary of Agriculture and Natural Resources
From the date of the approval of the application and for a term of five (5) for approval nor presented to the Registrar of Deeds in and for the
years from and after the date of issuance of the patent or grant, lands province of Cotabato for registration.
acquired under free patent or homestead provisions cannot be subject to
encumbrance or alienation, nor shall they become liable to the satisfaction Section 118 of Commonwealth Act No. 141 partly provides:
of any debt contracted prior to the expiration of said period. The only Except in favor of the Government or any of its branches, units, or
exception mentioned by the law is the sale or encumbrance in favor of the institutions, lands acquired under free patent or homestead provision shall
government or any of its branches, units or institutions. not be subject to encumbrance or alienation from the date of the approval
In a number of cases, we have consistently ruled that a sale of homestead of the application and for a term of five , years from and after the date of
within the five (5) year prohibitive period is void ab initio and the same cannot issuance of the patent or grant, nor shall they become liable to the
be ratified nor can it acquire validity through the passage of time. satisfaction of any debt contracted prior to the expiration of said period, but
the improvements or crops on the land may be mortgaged or pledged
to qualified persons, associations, or corporations.
Realty and the Bascaras/Jacinto. Marc Realty insists that Lee Chuy verbally The decision of respondent Court of Appeals is REVERSED and SET ASIDE.
notified of the sale and was given a copy of the deed of sale. The decision of the Regional Trial Court of Malolos, Bulacan is REINSTATED.
HELD:
The Court of Appeals erroneously concluded that a prior tender or offer of Ong Ching Po vs CA
redemption is a prerequisite or precondition to the filing of the action for FACTS:
legal redemption. To avail of the right of redemption what is essential is to Ong Joi Jong, sold a parcel of land located at Fundidor Street, San
make an offer to redeem within the prescribed period. There is no prescribed Nicolas to private respondent Soledad Parian on 23 July 1947. The
form for an offer to redeem to be properly effected. It can either be the sale was evidenced by a notarized Deed of Sale, written in English,
formal tender with consignation or the filing of a complaint in court. What is and was registered with the Register of Deeds of Manila which in
paramount is the availment of the fixed and definite period within which to turn issued TCT no. 9260 dated 2 September 1947 in the name of
exercise the right of legal redemption. private respondent. According to private respondent, she entrusted
the administration of the lot and building to Ong Ching Po when she
The filing of the action itself is equivalent to a formal offer to redeem. What and her husband settled in Iloilo, however when her husband died
constitutes a condition precedent is either a formal offer to redeem or the she demanded that the lot be vacated since she was going to sell it,
filing of an action in court together with the consignation of the redemption petitioners refused to vacate said premises. On 19 March 1984,
price within the reglementary period. Parian filed a case for unlawful detainer against Ong Ching Po. The
65 Tacder NatRes
MTC dismissed her case and was reaffirmed by both the RTC and also noteworthy that the tax receipts and rental receipts were in the name of
CA. Parian’s husband. Hence, petition was dismissed.
On the other hand, on 6 December 1983, Ong Ching Po executed a Frenzel vs Catito
Deed of Absolute Sale conveying the same property to his children DOCTRINE:
Jimmy and David Ong. On 12 December 1985, the Ong’s filed an A contract that violates the Constitution and the law, is null and void and
action for reconveyance and damages against Parian in RTC, Manila. vests no rights and creates no obligations. It produces no legal effect at
Upon the private respondent’s motion, this was consolidated with all. The petitioner, being a party to an illegal contract, cannot come into a
her action for quieting of title against the petitioners. The RTC court of law and ask to have his illegal objective carried out
rendered a decision in favor of private respondent and was
confirmed by the CA, hence this petition. FACTS: Petitioner Alfred Fritz Frenzel is an Australian citizen of German
descent. He was so enamored with Ederlina that he bought her numerous
ISSUE: Whether or not Ong Ching Po could legally acquire the property? properties such as house and lot in Quezon City and in Davao City. He also
put up a beauty parlor business in the name of Ederlina. Alfred was unaware
HELD: No. Petition is dismissed. that Ederlina was married until her spouse Klaus Muller wrote a letter to
Alfred begging the latter to leave her wife alone.
RATIO: Ong Ching Po cannot legally claim ownership of the disputed
property since the capacity to acquire private lands is dependent on the When Alfred and Ederlinas relationship started deteriorating. Ederlina had
capacity to acquire or hold lands of the public domain. Private land may be not been able to secure a divorce from Klaus. The latter could charge her for
transferred or conveyed only to individuals or entities “qualified to acquire bigamy and could even involve Alfred, who himself was still married. To avoid
lands of the public domain”. Petitioner Ong Ching Po was a Chinese citizen complications, Alfred decided to live separately from Ederlina and cut off all
and therefore is disqualified from acquiring and owning real property. contacts with her.
Further, despite the documentary evidence provided by petitioners, they
failed to provide evidence as to the genuineness and due execution of the On October 15, 1985, Alfred wrote to Ederlinas father, complaining that
deed of sale. Likewise, the tax receipts, tax declaration, rental receipts, deed Ederlina had taken all his life savings and because of this, he was virtually
of sale and TCT were in Ong’s possession, these were not reflective of penniless. He further accused the Catito family of acquiring for themselves
dominion or ownership as even a mere administrator or manager may the properties he had purchased with his own money. He demanded the
lawfully perform payment duties relative to his appointment as such. It was return of all the amounts that Ederlina and her family had stolen and turn
over all the properties acquired by him and Ederlina during their coverture.
66 Tacder NatRes
property, the court held that it was acquired using the paraphernal funds of
ISSUE: Whether the petitioner could recover the money used in purchasing Helmut, however, he cannot recover said property, nor have a right to
the several properties recover the funds used to buy it since it was purchased in violation of Section
7, Article XII of the Constitution. Upon appeal, the Court of Appeals granted
HELD: No, even if, as claimed by the petitioner, the sales in question were Helmut Muller’s prayer for reimbursement for the Antipolo property.
entered into by him as the real vendee, the said transactions are in violation
of the Constitution; hence, are null and void ab initio. A contract that violates ISSUE: WON Helmut Muller is entitled to reimbursement of the funds used
the Constitution and the law, is null and void and vests no rights and creates to acquire the Antipolo property.
no obligations. It produces no legal effect at all. The petitioner, being a party
to an illegal contract, cannot come into a court of law and ask to have his HELD: NO. Respondent was aware of the constitutional prohibition and
illegal objective carried out. One who loses his money or property by expressly admitted his knowledge thereof to this Court. He declared that he
knowingly engaging in a contract or transaction which involves his own moral had the Antipolo property titled in the name of petitioner because of the said
turpitude may not maintain an action for his losses. To him who moves in prohibition. His attempt at subsequently asserting or claiming a right on the
deliberation and premeditation, the law is unyielding. The law will not aid said property cannot be sustained. Thus, in the instant case, respondent
either party to an illegal contract or agreement; it leaves the parties where it cannot seek reimbursement on the ground of equity where it is clear that he
finds them willingly and knowingly bought the property despite the constitutional
prohibition. To allow reimbursement would in effect permit respondent to
enjoy the fruits of a property which he is not allowed to own. Thus, it is
Muller vs Muller likewise proscribed by law.
FACTS: Elena Buenaventura Muller and Helmut Muller are husband and wife
in this case. They wed and resided in Germany until they decided to Lee vs Director of Lands
permanently reside in the Philippines in 1992. By this time, they bought a FACTS:
house in Antipolo, Rizal using the proceeds that they got from selling the Sometime in March 1936, the Dinglasans sold to Lee Liong (Chinese
house the Helmut Muller inherited from his parents in Germany. citizen) a parcel of land situated at the corner of Roxas Avenue and
The marriage, however, did not last. They were eventually separated and Pavia Street, Roxas City.
Helmut Muller filed for separation of their properties. The trial court then In 1993, Elizabeth Manuel-Lee and Pacita Yu Lee filed with the RTC
dissolved the absolute community of property and ordered the equal of Roxas City a petition for reconstitution of title of the lot. (Alleging
partition of their personal properties located within the country, excluding that the transfer certificate of title issued to Lee Liong was lost or
those acquired by gratuitous title during marriage. As to the Antipolo destroyed during World War II.)
67 Tacder NatRes
Petitioners Elizabeth and Pacita alleged that they were the widows 2. WON the reconstitution was valid. NO
of the deceased Lee Bing Hoo and Lee Bun Ting, who were
the heirs of Lee Liong, the owner of the lot. HELD:
The RTC approved reconstitution of the lost or destroyed certificate 1. Lee Liong was not qualified but the ownership of the lot was
of title in the name of Lee Liong on the basis of an approved plan already acquired by Filipino citizens Lee Liong was disqualified to
and technical description. acquire the land under the 1935 Constitution. The sale of the land
Solicitor General filed with the Court of Appeals a petition for in question was consummated sometime in March 1936, during
annulment of the RTC decision alleging that the RTC had no the effectivity of the 1935 Constitution.
jurisdiction over the case.
The Solicitor General contended that the petitioners were not the Under the 1935 Constitution aliens could not acquire private agricultural
proper parties in the reconstitution of title, since their predecessor- lands, save in cases of hereditary succession. Thus, Lee Liong, a Chinese
in-interest Lee Liong did not acquire title to the lot because he was citizen, was disqualified to acquire the land in question.
a Chinese citizen and was constitutionally not qualified to own the
subject land. The fact that the Court did not annul the sale of the land to an alien did not
CA declared the reconstitution void. Hence this petition. validate the transaction. It was still contrary to the constitutional proscription
Elizabeth and Pacita emphasized that the ownership of the land had against aliens acquiring lands of the public or private domain.
been settled in two previous cases of the Supreme Court, where the
Court ruled in favor of their predecessor-in-interest, Lee Liong. The proper party to assail the sale is the Solicitor General.
They also pointed out that they acquired ownership of the land
through actual possession of the lot and their consistent payment of This was what was done in this case when the Solicitor General initiated
taxes over the land for more than sixty years. an action for annulment of judgment of reconstitution of title. While it
On the other hand, the Solicitor General submitted that the took the Republic more than sixty years to assert itself, it is not barred from
decision in the reconstitution case was void; otherwise, it would initiating such action. Prescription never lies against the State.
amount to circumventing the constitutional proscription against
aliens acquiring ownership of private or public agricultural lands. The land is now in the hands of Filipinos.
ISSUES: The original vendee, Lee Liong, has since died and the land has been
1. WON Lee Liong has the qualification to own land in the Philippines. inherited by his heirs and subsequently their heirs, petitioners herein.
NO Petitioners are Filipino citizens, a fact the Solicitor General does not dispute.
68 Tacder NatRes
does not pass upon the ownership of the land covered by the lost or
The constitutional proscription on alien ownership of lands of the public or destroyed title.
private domain was intended to protect lands from falling in the hands of
non-Filipinos. In this case, however, there would be no more public policy Any change in the ownership of the property must be the subject of a
violated since the land is in the hands of Filipinos qualified to acquire and separate suit. Thus, although petitioners are in possession of the land, a
own such land. separate proceeding is necessary to thresh out the issue of ownership of the
land.
“If land is invalidly transferred to an alien who subsequently becomes a
citizen or transfers it to a citizen, the flaw in the original transaction is 1.CONSTITUTIONAL LAW; PROSCRIPTION ON SALE OF REAL ESTATE TO
considered cured and the title of the transferee is rendered valid.” ALIENS; DOCTRINE OF PARI DELICTO APPLICABLE TO VENDOR AND
VENDEE THEREIN; CASE AT BAR. — "In sales of real estate to aliens incapable
Thus, the subsequent transfer of the property to qualified Filipinos may no of holding title thereto by virtue of the provisions of the Constitution both
longer be impugned on the basis of the invalidity of the initial transfer. The the vendor and the vendee are deemed to have committed the constitutional
objective of the constitutional provision to keep our lands in Filipino hands violation and being thus in pari delicto the courts will not afford protection
has been achieved. to either party." The proper party to assail the sale is the Solicitor General.
This was what was done in this case when the Solicitor General initiated an
Incidentally, it must be mentioned that reconstitution of the original action for annulment of judgment of reconstitution of title. While it took the
certificate of title must be based on an owner’s duplicate, secondary evidence Republic more than sixty years to assert itself, it is not barred from initiating
thereof, or other valid sources of the title to be reconstituted. such action. Prescription never lies against the State. Although ownership of
the land cannot revert to the original sellers, because of the doctrine of pari
2. Reconstitution was void for lack of factual support delicto, the Solicitor General may initiate an action for reversion or escheat
In this case, reconstitution was based on the plan and technical description of the land to the State, subject to other defenses, as hereafter set forth. In
approved by the Land Registration Authority. This renders the order of this case, subsequent circumstances militate against escheat proceedings
reconstitution void for lack of factual support. A judgment with absolutely because the land is now in the hands of Filipinos. The original vendee, Lee
nothing to support it is void. Liong, has since died and the land has been inherited by his heirs and
subsequently their heirs, petitioners herein. Petitioners are Filipino citizens, a
As earlier mentioned, a reconstitution of title is the re-issuance of a fact the Solicitor General does not dispute.
new certificate of title lost or destroyed in its original form and condition. It
69 Tacder NatRes
2.ID.; ID.; ID.; EFFECT OF SUBSEQUENT ACQUISITION OF PHILIPPINE Jose Ramirez a Filipino, died in Spain leaving only his widow Marcelle
CITIZENSHIP BY TRANSFEREE; CASE AT BAR. — The constitutional Ramirez, a French. In the project partition, the property was divided into 2
proscription on alien ownership of lands of the public or private domain was parts: 1st part to the widow, and 2nd part to the grandnephews the naked
intended to protect lands from falling in the hands of non-Filipinos. In this ownership. Furthermore, as to the usufruct of the 2 nd part, 1/3 was given to
case, however, there would be no more public policy violated since the land the widow and 2/3 to Wanda de Wrobleski, an Austrian. The grandnephews
is in the hands of Filipinos qualified to acquire and own such land. "If land is opposed on the ground that usufruct to Wanda is void because it violates
invalidly transferred to an alien who subsequently becomes a citizen or the constitutional prohibition against the acquisition of lands by aliens.
transfers it to a citizen, the flaw in the original transaction is considered cured
and the title of the transferee is rendered valid." Thus, the subsequent ISSUE:
transfer of the property to qualified Filipinos may no longer be impugned on
the basis of the invalidity of the initial transfer. The objective of the WON the ground for the opposition is correct.
constitutional provision to keep our lands in Filipino hands has been
achieved. ASCTac HELD:
3.CIVIL LAW; LAND REGISTRATION; RECONSTITUTION OF TITLE; VALID No, it is not correct.
SOURCES OF THE TITLE TO BE RECONSTITUTED; CASE AT BAR. —
Incidentally, it must be mentioned that reconstitution of the original The SC held that the Constitutional provision which enables aliens to
certificate of title must be based on an owner's duplicate, secondary evidence acquire private lands does not extend to testamentary succession for
thereof, or other valid sources of the title to be reconstituted. In this case, otherwise the prohibition will be for naught and meaningless. The SC upheld
reconstitution was based on the plan and technical description approved by the usufruct in favor of Wanda because although it is a real right, it does not
the Land Registration Authority. This renders the order of reconstitution void vest title to the land in the usufructuary and it is the vesting of title to land in
for lack of factual support. A judgment with absolutely nothing to support it favor of aliens which is proscribed by the Constitution.
is void.
Republic vs CA
On June 17, 1978, respondent spouses bought Lots, as their residence with a
total area of 91.77 sq. m. situated in San Pablo City, from one Cristeta Dazo
Ramirez vs Vda de Ramirez Belen. At the time of the purchase, respondent spouses where then natural-
FACTS: born Filipino citizens.
70 Tacder NatRes
On February 5, 1987, the spouses filed an application for registration of title land, such possession ipso jure converts the same to private property
of the two (2) parcels of land before the RTC of San Pablo City. This time, (Recall ruling of Director of Lands v. IA C). This means that occupation and
however, they were no longer Filipino citizens and have opted to embrace cultivation for more than 30 years by an applicant and his predecessors-in-
Canadian citizenship through naturalization. interest, vest title on such applicant so as to segregate the land from the
mass of public.
RTC rendered a decision confirming the title of the Spouses. Republic filed
an opposition to the decision of RTC. On appeal, CA affirmed the decision of What is important is that private respondents were formerly natural-born
RTC. citizens of the Philippines, and as transferees of a private land, they could
apply for registration in accordance with the mandate of Section 8, Article XII
Republic submits that private respondents have not acquired proprietary of the Constitution.
rights over the subject properties before they acquired Canadian citizenship
through naturalization to justify the registration thereof in their favor. It In the case at bar, the spouses were undoubtedly natural-born
maintains that even privately owned unregistered lands are presumed to be Filipino citizens at the time of the acquisition of the properties and by virtue
public lands under the principle that lands of whatever classification belong thereof, acquired vested rights thereon, tacking in the process, the
to the State under the Regalian doctrine. Thus, before the issuance of the possession in the concept of owner and the prescribed period of time held
certificate of title, the occupant is not in the juridical sense the true owner of by their predecessors-in-interest under the Public Land Act.
the land since it still pertains to the State. Petitioner further argued that it is
only when the court adjudicates the land to the applicant for confirmation of But what should not be missed in the disposition of this case is the
title would the land become privately owned land, for in the same fact that the Constitution itself allows private respondents to register the
proceeding, the court may declare it public land, depending on the evidence. contested parcels of land in their favor. Sections 7 and 8 of Article XII of the
Constitution contain the following pertinent provisions, to wit:
ISSUE: WON a foreign national apply for registration of title over a parcel of
land which he acquired by purchase while still a citizen of the Philippines, Sec. 8. Notwithstanding the provisions of Section 7 of this Article, a natural-
from a vendor who has complied with the requirements for registration born citizen of the Philippines who has lost his Philippine citizenship may be
under the Public Land Act (CA 141)? YES a transferee of private lands, subject to limitations provided by law .
(Emphasis supplied)
HELD:
Since the spouses’ predecessor in interest has been in open, Section 8, Article XII of the 1987 Constitution above quoted is similar to
continuous and exclusive possession for at least 30 years of alienable public Section 15, Article XIV of the then 1973 Constitution which reads:
71 Tacder NatRes
Even if private respondents were already Canadian citizens at the time they
applied for registration of the properties in question, said properties as
72 Tacder NatRes
Grounds
land covered thereby is a forest or timber land which is not
disposable under the Public Land Act;
in a reclassification of the public lands in the vicinity of the land in
question made by the Bureau of Forestry on March 7, 1958, was
reverted to the category of public forest,
the application for free patent by Isagani Du Timbol more than
Who can initiate eleven years thereafter were obtained fraudulently as private
Republic vs Animas respondent Isagani Du Timbol never occupied and cultivated the
ORIGINAL CASE: Republic of the Philippines, Plaintiff, vs. Isagani Du Timbol land applied for.
and the Register of Deeds of General Santos City, RTC -dismissed the complaint on the ground that Certificate of Title based
NOW Director of lands instituted to on the patent had became indefeasible in view of the lapse of the one-year
1. declare null and void Free Patent issued in the name of defendant period prescribed under Section 38 of the Land Registration Act for review
Isagani Du Timbol; of a decree of title on the ground of fraud.
2. to order the aforesaid defendant to surrender the owner's duplicate CA- grants the petition on the ground that the area covered by the patent
of O.C.T. and the and title is not a disposable public land, it being a part of the forest zone
3. defendant Register of Deeds to cancel the same; and, hence the patent and title thereto are null and void.
4. to decree the reversion of the land in question to the mass of public
domain Issue:WON LAND CAN BE REVERTED BACK TO THE STATE
The land was originally applied for by Precila Soriain 1966, who transferred
her rights to the land and its improvements to defendant Isagani Du Timbol Held: YES. A certificate of title fraudulently secured is null and void ab initio
who filed his application on1969, when fraud consisted in misrepresenting that the land covered by the
In 1969, free Patent No. was issued and City, (O.C.T.) No. P-2508 in the name application is part of the public domain when it is not
of defendant Isagani Du Timbol.
73 Tacder NatRes
As a general rule, timber or forest lands are not alienable or disposable under The defense of indefeasibility of a certificate of title issued pursuant to a free
either the Constitution of 1935 or the Constitution of 1973. patent does not lie against the state in an action for reversion of the land
covered thereby when such land is a part of a public forest or of a forest
When the defendant Isagani Du Timbol filed his application for free patent reservation.
over the land in question on June 3, 1969, the area in question was not a The lapse of the one year period within which a decree of title may be
disposable or alienable public land but a public forest. reopened for fraud would not prevent the cancellation thereof, for to hold
It is the Bureau of Forestry that has jurisdiction and authority over the that a title may become indefeasible by registration, even if such title had
demarcation, protection, management, reproduction, occupancy and use of been secured through fraud or in violation of the law, would be the height
all public forests and forest reservations and over the granting of licenses for of absurdity. Registration should not be a shield of fraud in securing title.
the taking of products therefrom, including stone and earth. Given that the
area in question is a forest or timber land is clearly established, then, the It is the state is seeking the cancellation of the title, therefore, it has not
Bureau of Forest Development has jurisdiction. become indefeasible for prescription cannot be invoked against the state. A
title founded on fraud may be cancelled, notwithstanding the lapse of one
Since it was Bureau of Lands who issued the patent and since it had no year from the issuance thereof, through a petition filed in court by the
jurisdiction to issue a patent because the land involved was still inalienable Solicitor General.
forest land when granted, then it may be plausibly contended that the patent
title would be ab initio void, subject to attack at any time by any party The land covered thereby may be reconveyed to the state in an action for
adversely affected. reconveyance under Section 101 of Commonwealth Act 141 (Public Land Act).
A patent is void at law if the
officer who issued the patent had no authority to do so Isagani Du Timbol was never in possession of the property prior to his filing
If a person obtains a title by FAME under the Torrens System the application, contrary to the provisions of law that
Director of Lands did not have jurisdiction over the same because it the applicant must have been in possession or cultivation thereof for
is a public forest, the grantee does not, by virtue of said certificate at least 30 years;
of title alone, become the owner of the land illegally included. no existing signs of improvements found in the area in question as
In the case at bar the party seeking the nullity of the title and reversion of it is not under cultivation but covered with grasses, bushes and small
the land is the state itself which is specifically authorized under Section 101 trees;
of the Public Land Act to initiate such proceedings as an attribute of that it is being used as ranch for grazing cows by the heirs of
sovereignty. Hermogenes Chilsot;
74 Tacder NatRes
that no monuments were placed on the area surveyed which goes with the Bureau of Lands to support their claim that they were entitled to the
to show that there was no actual survey thereof; issuance of a certificate of title over the said land on which they said they had
inside the forest zone; already made full payment. On the basis of this affidavit, the Secretary of
the signature of then Acting District Land Officer Elias de Castro of Agriculture and Natural Resources executed Deed No. V-10910 (Sale
South Cotabato has been forged Certificate No. 1280) on September 10, 1971, in favor of the said affiants.
The above alleged circumstances are indicative of fraud and the
misrepresentations of the applicant that he had been occupying and A complaint for reversion was filed on October 10, 1985 when the
cultivating the land and residing thereon are sufficient grounds to ipso facto registered owners of the land, following several transfers, were Remedios
nullify the grant of the patent and title under Section 91 of the Public Land Micla, Juan C. Pulido, and Rosalina, Luz and Enrique Naval. They asked to
Law which provides as follows: return the property to the State on the aforestated grounds of forgery and
A certificate of title that is void may be ordered cancelled. fraud. The plaintiff claimed that Gregorio Cenizal having died on February
A title will be considered void if it is procured through fraud, as when a 25, 1943, and Maria Cenizal on January 8, 1959, they could not have signed
person applies for registration of the land under his name although the the joint affidavit dated August 9, 1971, on which Deed No. V-10910 (Sale
property belongs to another. Certificate No. 1280) was based.
In their answer, Pulido and the Navals denied any participation in the
join affidavit and said they had all acquired the property in good faith and
When proper/ improper for value. By way of affirmative defenses, they invoked estoppel, laches,
prescription and res judicata. For her part, Miclat moved to dismiss the
complaint, contending that the government had no cause of action against
Republic vs Umali her because there was no allegation that she had violated the plaintiff’s right,
FACTS that the government was not the real party-in-interest because the subject
land was already covered by the Torrens system, and that in any event the
The land situated in Tanza, Cavite which consists of 78,865 square action was barred by prescription or laches.
meters was originally purchased on installment from the government on July
1, 1910 by Florentina Bobadilla, who allegedly transferred her rights thereto
in favor of Martina, Tomasa, Gregorio and Julio, all surnamed Cenizal, in 1922. ISSUE
2 Tomasa and Julio assigned their shares to Martina, Maria and Gregorio. In
1971 these three assignees purportedly signed a joint affidavit which was filed
75 Tacder NatRes
Whether or not the land under the new owners are obtained thru forgery evidence to the contrary. The petitioner contends that it was Pedro Miclat
and fraud and subject to return the property to the State who caused the falsification of the joint affidavit, but that is a bare and hardly
persuasive allegation, and indeed, even if true, would still not prove any
HELD collusion between him and the private respondents. The mere fact that
Remedios Miclat was the daughter and heiress of Miclat, without more,
We agree with the contention that there is no allegation in the would not necessarily visit upon her the alleged sins of her father.
complaint 13 filed by the petitioner that any one of the defendants was privy The land being now registered under the Torrens system in the
to the forged joint affidavit or that they had acquired the subject land in bad names of the private respondents, the government has no more control or
faith. Their status as innocent transferees for value was never questioned in jurisdiction over it. It is no longer part of the public domain or, as the Solicitor
that pleading. Not having been disproved, that status now accords to them General contends — as if it made any difference — of the Friar Lands. The
the protection of the Torrens System and renders the titles obtained by them subject property ceased to be public land when OCT No. 180 was issued to
thereunder indefeasible and conclusive. The rule will not change despite the Florentina Bobadilla in 1910 or at the latest from the date it was sold to the
flaw in TCT No. 55044. Cenizals in 1971 upon full payment of the purchase price. As private
Section 39 of the Land Registration Act clearly provided: Every registered land, it is governed by the provisions of the Land Registration Act,
person receiving a certificate of title in pursuance of a decree of registration, now denominated the Property Registration Decree, which applies even to
and every subsequent purchaser of registered land who takes a certificate of the government.
title for value in good faith shall hold the same free of all encumbrance except
those noted on said certificate. 1. LAND REGISTRATION; TORRENS SYSTEM; PROTECTS INNOCENT
The real purpose of the Torrens System of land registration is to TRANSFEREES FOR VALUE; TITLES OBTAINED RENDERED INDEFEASIBLE
quiet title to land; to put a stop forever to any question of the legality of the AND CONCLUSIVE. — There is no allegation in the complaint filed by the
title, except claims which were noted at the time of registration in the petitioner that any one of the defendants was privy to the forged joint
certificate, or which may arise subsequent thereto. That being the purpose affidavit or that they had acquired the subject land in bad faith. Their status
of the law, it would seem that once the title was registered, the owner might as innocent transferees for value was never questioned in that pleading. Not
rest secure, without the necessity of waiting in the portals of the court, or having been disproved, that status now accords to them the protection of
sitting in the "mirador de su casa," to avoid the possibility of losing his land. the Torrens System and renders the titles obtained by them thereunder
The difference between them and the private respondents is that the indefeasible and conclusive. The rule will not change despite the flaw in TCT
latter acquired the land in question not by direct grant but in fact after several No. 55044. Section 39 of the Land Registration Act clearly provided: "Sec. 39.
transfers following the original sale thereof to Bobadilla in 1910. The Every person receiving a certificate of title in pursuance of a decree of
presumption is that they are innocent transferees for value in the absence of registration, and every subsequent purchaser of registered land who takes a
76 Tacder NatRes
certificate of title for value in good faith shall hold the same free of all now denominated the Property Registration Decree, which applies even to
encumbrance except those noted on said certificate." the government. The pertinent provision of the Land Registration Act was
Section 122, which read as follows: Sec. 122. "Whenever public lands in the
2. ID.; ID.; ID.; PIÑERO v. DIRECTOR OF LANDS, 57 SCRA 386, NOT Philippine Islands belonging to the Government of the United States or to
APPLICABLE IN CASE AT BAR. — The decision in Piñero v. Director of Lands the Government of the Philippine Islands are alienated, granted, or conveyed
is not applicable to the present proceeding because the lands involved in to persons or to public or private corporations, the same shall be brought
that case had not yet passed to the hands of an innocent purchaser for value. forthwith under the operation of this Act and shall become registered lands."
They were still held by the Piñeros. The action for reversion was filed by the This should be related to Section 12 of the Friar Lands Act, providing thus:
government against them as the original transferees of the properties in "Sec. 12. . . . upon the payment of the final installment together with all
question. They were the direct grantees of the free patents issued by the accrued interest, the Government will convey to such settler and occupant
government pursuant to which the corresponding certificates of title were the said land so held by him by proper instrument of conveyance, which shall
issued under the Torrens system. The fraud alleged by the government as a be issued and become effective in the manner provided in section one
ground for the reversion sought was imputable directly to the Piñeros, who hundred and twenty-two (Sec. 122) of the Land Registration Act."
could not plead the status of innocent purchasers for value.The difference
between them and the private respondents is that the latter acquired the 4. ID.; ID.; ORIGINAL TRANSFER OF LAND, MERELY AVOIDABLE, NOT VOID
land in question not by direct grant but in fact after several transfers following AB-INITIO; LAND CEASES TO BE PUBLIC UPON REGISTRATION AND
the original sale thereof to Bobadilla in 1910. The presumption is that they ISSUANCE OF CERTIFICATE OF TITLE. — The petitioner errs in arguing that
are innocent transferees for value in the absence of evidence to the contrary. the original transfer was null and void ab initio, for the fact is that it is not so.
It was only voidable. The land remained private as long as the title thereto
3. ID.; ID.; SUBJECT LAND NO LONGER PART OF PUBLIC DOMAIN; LAND had not been voided, but it is too late to do that now. As the Court has held
REGISTRATION ACT, NOW CALLED "PRIVATE REGISTRATION DECREE" in Ramirez vs. Court of Appeals. (30 SCRA 301): "A certificate of title
GOVERNS. — The land being now registered under the Torrens system in fraudulently secured is not null and void ab initio, unless the fraud consisted
the names of the private respondents, the government has no more control in misrepresenting that the land is part of the public domain, although it is
or jurisdiction over it. It is no longer part of the public domain or, as the not. In such case the nullity arises, not from the fraud or deceit, but from the
Solicitor General contends — as if it made any difference — of the Friar fact that the land is not under the jurisdiction of the Bureau of Lands.
Lands. The subject property ceased to be public land when OCT No. 180 was Inasmuch as the land involved in the present case does not belong to such
issued to Florentina Bobadilla in 1910 or at the latest from the date it was sold category. OCT No. 282-A would be merely voidable or reviewable. . . . Once
to the Cenizals in 1971 upon full payment of the purchase price. As private a patent is registered and the corresponding certificate of title is issued, the
registered land, it is governed by the provisions of the Land Registration Act, land ceases to be part of public domain and becomes private property over
77 Tacder NatRes
which the director of Lands has neither control nor jurisdiction. A public land
patent, when registered in the corresponding Register of Deeds, is a veritable ISSUE:
Torrens Title, and becomes as indefeasible as Torrens Title upon the Whether or not the actual occupancy of a part of the land described in the
expiration of one (1) year from the date of issuance thereof." instrument giving color of title sufficient to give title to the entire tract of
land?
HELD:
The general rule is that possession and cultivation of a portion of a tract of
Pinero vs Director of Lands land under the claim of ownership of all is a constructive possession of all, if
The general rule is that possession and cultivation of a portion of a tract of the remainder is not in the adverse possession of another.
land under the claim of ownership of all is a constructive possession of all, if
the remainder is not in the adverse possession of another. The claimant has color of title; he acted in good faith and he has open,
peaceable, and notorious possession of a portion of the property, sufficient
FACTS: to apprise the community and the world that the land was for his enjoyment.
Restituo Romero gained possession of a considerable tract of land located
in Nueva Ecija. He took advantage of the Royal Decree to obtain a possessory Possession in the eyes of the law does not mean that a man has to have his
information title to the land and was registered as such. feet on every square meter of ground before it can be said that he is in
possession. Ramos and his predecessor in interest fulfilled the requirements
Parcel No. 1 included within the limits of the possessory information title of of the law on supposition that the premises consisted of agricultural public
Romero was sold to Cornelio Ramos, herein petitioner. land.
Ramos instituted appropriate proceedings to have his title registered. On the issue of forest land, Forest reserves of public land can be established
Director of Lands opposed on the ground that Ramos had not acquired a as provided by law. When the claim of the citizen and the claim of the
good title from the Spanish government. government as to a particular piece of property collide, if the Government
desires to demonstrate that the land is in reality a forest, the Director of
Director of Forestry also opposed on the ground that the first parcel of land Forestry should submit to the court convincing proof that the land is not
is forest land. more valuable for agricultural than for forest purposes.
It has been seen however that the predecessor in interest to the petitioner at
least held this tract of land under color of title. In this case, the mere formal opposition on the part of the Attorney-General
78 Tacder NatRes
Evangelista vs Santiago
Roco vs Gineda
Republic vs Hachero
Republic vs Espinosa